Метод гаусса решение систем линейных уравнений: Онлайн калькулятор. Решение систем линейных уравнений. Метод Гаусса.

Содержание

принцип, теорема и примеры решения задач

Задание. Решить СЛАУ $\left\{\begin{array}{l} 2 x_{1}+x_{2}+x_{3}=2 \\ x_{1}-x_{2}=-2 \\ 3 x_{1}-x_{2}+2 x_{3}=2 \end{array}\right.$ методом Гаусса.

Решение. Выпишем расширенную матрицу системы и при помощи элементарных преобразований над ее строками приведем эту матрицу к ступенчатому виду (прямой ход) и далее выполним обратный ход метода Гаусса (сделаем нули выше главной диагонали). Вначале поменяем первую и вторую строку, чтобы элемент $a_{11}$ равнялся 1 (это мы делаем для упрощения вычислений):

$$\tilde{A}=A \mid B=\left(\begin{array}{rrr|r} 2 & 1 & 1 & 2 \\ 1 & -1 & 0 & -2 \\ 3 & -1 & 2 & 2 \end{array}\right) \sim\left(\begin{array}{rrr|r} 1 & -1 & 0 & -2 \\ 2 & 1 & 1 & 2 \\ 3 & -1 & 2 & 2 \end{array}\right)$$

Далее делаем нули под главной диагональю в первом столбце.

Для этого от второй строки отнимаем две первых, от третьей — три первых:

$$\tilde{A} \sim\left(\begin{array}{rrr|r} 1 & -1 & 0 & -2 \\ 0 & 3 & 1 & 6 \\ 0 & 2 & 2 & 8 \end{array}\right)$$

Все элементы третьей строки делим на два (или, что тоже самое, умножаем на $\frac{1}{2}$ ):

$$\tilde{A} \sim\left(\begin{array}{rrr|r} 1 & -1 & 0 & -2 \\ 0 & 3 & 1 & 6 \\ 0 & 1 & 1 & 4 \end{array}\right)$$

Далее делаем нули во втором столбце под главной диагональю, для удобства вычислений поменяем местами вторую и третью строки, чтобы диагональный элемент равнялся 1:

$$\tilde{A} \sim\left(\begin{array}{ccc|c} 1 & -1 & 0 & -2 \\ 0 & 1 & 1 & 4 \\ 0 & 3 & 1 & 6 \end{array}\right)$$

От третьей строки отнимаем вторую, умноженную на 3:

$$\tilde{A} \sim\left(\begin{array}{rrr|r} 1 & -1 & 0 & -2 \\ 0 & 1 & 1 & 4 \\ 0 & 0 & -2 & -6 \end{array}\right)$$

Умножив третью строку на $\left(-\frac{1}{2}\right)$ , получаем:

$$\tilde{A} \sim\left(\begin{array}{rrr|r} 1 & -1 & 0 & -2 \\ 0 & 1 & 1 & 4 \\ 0 & 0 & 1 & 3 \end{array}\right)$$

Проведем теперь обратный ход метода Гаусса (метод Гассу-Жордана), то есть сделаем нули над главной диагональю.

Начнем с элементов третьего столбца. Надо обнулить элемент $a_{23}$, для этого от второй строки отнимем третью:

$$\tilde{A} \sim\left(\begin{array}{rrr|r} 1 & -1 & 0 & -2 \\ 0 & 1 & 0 & 1 \\ 0 & 0 & 1 & 3 \end{array}\right)$$

Далее обнуляем недиагональные элементы второго столбца, к первой строке прибавляем вторую:

$$\tilde{A} \sim\left(\begin{array}{ccc|c} 1 & 0 & 0 & -1 \\ 0 & 1 & 0 & 1 \\ 0 & 0 & 1 & 3 \end{array}\right)$$

Полученной матрице соответствует система

$\left\{\begin{array}{l}x_{1}+0 \cdot x_{2}+0 \cdot x_{3}=-1 \\ 0 \cdot x_{1}+x_{2}+0 \cdot x_{3}=1 \\ 0 \cdot x_{1}+0 \cdot x_{2}+x_{3}=3\end{array}\right.$    или   $\left\{\begin{array}{l} x_{1}=-1 \\ x_{2}=1 \\ x_{3}=3 \end{array}\right.$

Ответ. $\left\{\begin{array}{l} x_{1}=-1 \\ x_{2}=1 \\ x_{3}=3 \end{array}\right.$

Метод Гаусса — примеры c решением, теоремы и формулы

Карл Фридрих Гаусс – немецкий математик, основатель одноименного метода решения СЛАУ

Карл Фридрих Гаусс – был известным великим математиком и его в своё время признали «королём математики». Хотя название «метод Гаусса» является общепринятым, Гаусс не является его автором: метод Гаусса был известен задолго до него. Первое его описание имеется в китайском трактате «Математика в девяти книгах», который составлен между II в. до н. э. и I в. н. э. и представляет собой компиляцию более ранних трудов, написанных примерно в X в. до н. э.

Метод Гаусса – последовательное исключение неизвестных. Этот метод используется для решения квадратных систем линейных алгебраических уравнений. Хотя уравнения при помощи метода Гаусса решаются легко, но всё же студенты часто не могут найти правильное решение, так как путаются в знаках (плюсы и минусы). Поэтому во время решения СЛАУ необходимо быть предельно внимательным и только тогда можно легко, быстро и правильно решить даже самое сложное уравнение.

У систем линейных алгебраических уравнений есть несколько преимуществ: уравнение не обязательно заранее на совместность; можно решать такие системы уравнений, в которых число уравнений не совпадает с количеством неизвестных переменных или определитель основной матрицы равняется нулю; есть возможность при помощи метода Гаусса приводить к результату при сравнительно небольшом количестве вычислительных операций.

Определения и обозначения

Как уже говорилось, метод Гаусса вызывает у студентов некоторые сложности. Однако, если выучить методику и алгоритм решения, сразу же приходит понимание в тонкостях решения.

Для начала систематизируем знания о системах линейных уравнений.

Обратите внимание!

СЛАУ в зависимости от её элементов может иметь:

  1. Одно решение;
  2. много решений;
  3. совсем не иметь решений.

В первых двух случаях СЛАУ называется совместимой, а в третьем случае – несовместима. Если система имеет одно решение, она называется определённой, а если решений больше одного, тогда система называется неопределённой.

Метод Крамера и матричный способ не подходят для решения уравнений, если система имеет бесконечное множество решений. Вот поэтому нам и нужен метод Гаусса, который поможет нам в любом случае найти правильное решение. К элементарным преобразованиям относятся:

  • перемена мест уравнений системы;
  • почленное умножение обеих частей на одно из уравнений на некоторое число, так, чтобы коэффициенты при первой переменной в двух уравнениях были противоположными числами;
  • сложение к обеим частям одного из уравнений определённых частей другого уравнения.

Итак, когда мы знаем основные правила и обозначения, можно приступать к решению.

Теперь рассмотрим, как решаются системы методом Гаусса на простом примере:

   

где а, в, с  – заданные коэффициенты, d – заданные свободные члены, x, y, z – неизвестные. Коэффициенты и свободные члены уравнения можно называть его элементами.

Если = = = , тогда система линейных алгебраических уравнений называется однородной, в другом случае – неоднородной.

Множественные числа , , называются решением СЛАУ, если при подстановке , , в СЛАУ получим числовые тождества.

Система, которую мы написали выше имеет координатную форму. Если её переделать в матричную форму, тогда система будет выглядеть так:

– это основная матрица СЛАУ.

– матрица столбец неизвестных переменных.

– матрица столбец свободных членов.

Если к основной матрице добавить в качестве – ого столбца матрицу-столбец свободных членов, тогда получится расширенная матрица систем линейных уравнений.

Как правило, расширенная матрица обозначается буквой , а столбец свободных членов желательно отделить вертикальной линией от остальных столбцов. То есть, расширенная матрица выглядит так:

Если квадратная матрица равна нулю, она называется вырожденная, а если – матрица невырожденная.

Обратите внимание!

Если с системой уравнений:          

Произвести такие действия:

  • умножать обе части любого из уравнений на произвольное и отличное от нуля число ;
  • менять местами уравнения;
  • к обеим частям любого из уравнений прибавить определённые части другого уравнения, которые умножаются на произвольное число ,

тогда получается эквивалентная система, у которой такое же решение или нет решений совсем.

Теперь можно перейти непосредственно к методу Гаусса.

Нужна помощь в написании работы?

Мы — биржа профессиональных авторов (преподавателей и доцентов вузов). Наша система гарантирует сдачу работы к сроку без плагиата. Правки вносим бесплатно.

Цена работы

Простейшие преобразования элементов матрицы

Мы рассмотрели основные определения и уже понимаем, чем нам поможет метод Гаусса в решении системы. Теперь давайте рассмотрим простую систему уравнений. Для этого возьмём самое обычное уравнение, где и используем решение методом Гаусса:

Из уравнения запишем расширенную матрицу:

Из данной матрицы видно, по какому принципу она записана. Вертикальную черту не обязательно ставить, но просто так удобнее решать систему.

Определение

Матрица системы – это матрица, которая составляется исключительно с коэффициентами при неизвестных. Что касается расширенной матрицы системы, так, это такая матрица, в которой кроме коэффициентов записаны ещё и свободные члены. Любую из этих матриц называют просто матрицей.

На матрице, которая написана выше рассмотрим, какие существуют элементарные преобразования:

1. В матрице строки можно переставлять местами. Например, в нашей матрице спокойно можно переставить первую и вторую строки:

.

2. Если в матрице имеются (или появились) пропорциональные строки (одинаковые), тогда необходимо оставить всего лишь одну строку, а остальные убрать (удалить).

3. Если в ходе преобразований в матрице появилась строка, где находятся одни нули, тогда такую строку тоже нужно удалять.

4. Строку матрицы можно умножать (делить) на любое число, которое отличное от нуля. Такое действие желательно проделывать, так как в будущем проще преобразовывать матрицу.

5. Сейчас рассмотрим преобразование, которое больше всего вызывает затруднение у студентов. Для этого возьмём изначальную нашу матрицу:

Для удобства умножаем первую строку на (-3):

Теперь ко второй строке прибавляем первую строку, которую умножали на -3. Вот что у нас получается:

В итоге получилось такое преобразование:

Теперь для проверки можно разделить все коэффициенты первой строки на те же и вот что получается:

В матрице верхняя строка преобразовалась:

Первую строку делим на и преобразовалась нижняя строка:

И верхнюю строку поделили на то же самое число :

Как вы можете убедиться, в итоге строка, которую мы прибавляли ни капельки не изменилась, а вот вторая строка поменялась. ВСЕГДА меняется только та строка, к которой прибавляются коэффициенты.

Мы расписали в таких подробностях, чтобы было вам понятно, откуда какая цифра взялась. На практике, например, на контрольной или экзамене матрица так подробно не расписывается. Как правило, в задании решение матрицы оформляется так:

.

Обратите внимание!

Если в примере приведены десятичные дроби, метод Гаусса в этом случае также поможет решить систему линейных алгебраических уравнений. Однако, не стоит забывать, что следует избегать приближённых вычислений, так как ответ будет неверным. Лучше всего использовать десятичные дроби, а от них переходить к обыкновенным дробям.

Алгоритм решения методом Гаусса пошагово

После того, как мы рассмотрели простейшие преобразования, в которых на помощь пришёл метод Гаусса, можем вернуться к нашей системе, которую уже разложили по полочкам и пошагово распишем:

Шаг 1. Переписываем систему в виде матрицы

Записываем матрицу:

Шаг 2.

Преобразовываем матрицу: вторую строку в первом столбце приводим к нулю

Как мы привели вторую строку в первом столбце к нулю описано выше. Напомним, что первую строку умножали на и вторую строку прибавили к первой , умноженной на .

Шаг 3. Приводим матрицу к ступенчатому виду

Теперь вторую строку можно поделить на 2 и получается:

Верхнюю строку делим на и приводим матрицу к ступенчатому виду:

Когда оформляют задание, так и отчёркивают простым карандашом для упрощения работы, а также обводят те числа, которые стоят на “ступеньках”. Хотя в учебниках и другой литературе нет такого понятия, как ступенчатый вид. Как правило, математики такой вид называют трапециевидным или треугольным.

Шаг 4. Записываем эквивалентную систему

После наших элементарных преобразований получилась эквивалентная система:

Шаг 5. Производим проверку (решение системы обратным путём)

Теперь систему нужно решить в обратном направлении, то есть обратным ходом, начиная с последней строки. :

находим : ,

,

.

После находим :

,

.

Тогда:

.

Как видим, уравнение решено правильно, так как ответы в системе совпадают.

Решение систем линейных уравнений методом Гаусса, в которых основная матрица невырожденная, а количество в ней неизвестных равняется количеству уравнений

Как мы уже упоминали, невырожденная матрица бывает тогда, когда . Разберём систему уравнений невырожденной матрицы, где уравнений по количеству столько же, сколько и неизвестных. Эту систему уравнений решим другим способом.

Дана система уравнений:

Для начала нужно решить первое уравнение системы относительно неизвестной переменной . Далее подставим полученное выражение сначала во второе уравнение, а затем в третье, чтобы исключить из них эту переменную.

Теперь переходим ко второму уравнению системы относительно и полученный результат подставим в третье уравнение.. Это нужно для того, чтобы исключить неизвестную переменную :

Из последнего, третьего уравнения мы видим, что . Из второго уравнения находим . И последнее, находим первое уравнение .

Итак, мы нашли все три неизвестных при помощи последовательного исключения. Такой процесс называют – прямой ход метода Гаусса. Когда последовательно находятся неизвестные переменные, начиная с последнего уравнения, называется обратным ходом метода Гаусса.

Когда выражается через и в первом уравнении, а затем подставляется полученное выражение во второе или третье уравнения, тогда, чтобы привести в к такому же результату, необходимо проделать такие действия:

  • берём второе уравнение и к его левой и правой частям прибавляем определённые части из первого уравнения, которые умножаются на ,
  • берём третье уравнение и к его левой и правой частям прибавляем определённые части из первого уравнения, которые умножаются на .

И действительно, благодаря такой процедуре у нас есть возможность исключать неизвестную переменную со второго и третьего уравнения системы:

Возникают нюансы с исключением неизвестных переменных тогда, когда в уравнении системы нет каких-либо неизвестных переменных. Рассмотрим такую систему:

В этой системе в первом уравнении нет переменной и поэтому у нас нет возможности решить первое уравнение системы относительно , чтобы исключить данную переменную из остальных уравнений. В таком случае выход есть. Нужно всего лишь уравнения переставить местами.

Так как мы описываем уравнения системы, в которых определитель основных матриц отличен от нуля, тогда всегда есть такое уравнение, в котором есть необходимая нам переменная и это уравнение мы можем поставить туда, куда нам нужно.

В примере, который мы рассматриваем, достаточно всего лишь поменять местами первое и второе уравнение.

Теперь мы можем спокойно разрешить первое уравнение относительно переменной и убрать (исключить) из остальных уравнений в системе. Вот и весь принцип работы с такими, на первый взгляд, сложными системами.

Решение систем линейных уравнений методом Гаусса, в которых основная матрица вырожденная, а количество в ней неизвестных не совпадает с количеством уравнений

Метод Гаусса помогает решать системы уравнений, у которых основная матрица прямоугольная или квадратная, но основная вырожденная матрица может совсем не иметь решений, иметь бесконечное множество решений или иметь всего лишь одно единственное решение.

Рассмотрим, как при помощи метода Гаусса устанавливается совместность или несовместность систем линейных уравнений. В случае, если есть совместность определим все решения или одно решение.

В принципе, исключать неизвестные переменные можно точно так, как описано выше. Однако, есть некоторые непонятные ситуации, которые могут возникнуть в ходе решения:

1. На некоторых этапах в момент исключения неизвестных переменных некоторые уравнения могут обратиться в тождества . В данном случае такие уравнения лишние в системе и их можно смело полностью убирать, а затем продолжать решать уравнение методом Гаусса.

Например, вам попалась подобная система:

У нас получается такая ситуация

Как видим, второе уравнение . Соответственно, данное уравнение мы можем из системы удалить, так как оно без надобности.

Дальше можно продолжать решение системы линейных алгебраических уравнений уравнений традиционным методом Гаусса.

2. При решении уравнений прямым ходом методом Гаусса могут принять не только одно, но и несколько уравнений такой вид: , где – число, которое отличное от нуля. Это говорит о том, что такое уравнение никогда не сможет превратиться в тождество даже при любых значениях неизвестных переменных. То есть, можно выразить по-другому. Если уравнение приняло  вид, значит система несовместна, то есть, не имеет решений. Рассмотрим на примере:

Для начала необходимо исключить неизвестную переменную из всех уравнений данной системы, начиная со второго уравнения. Для этого нужно прибавить к левой и правой частям второго, третьего, четвёртого уравнения части (левую и правую) первого уравнения, которые соответственно, умножаются на (-1), (-2), (-3). Получается:

В третьем уравнении получилось равенство . Оно не подходит ни для каких значений неизвестных переменных , и , и поэтому, у данной системы нет решений. То есть, говорится, что система не имеет решений.

3. Допустим, что при выполнении прямого хода методом Гаусса нам нужно исключить неизвестную переменную , и ранее, на каком-то этапе у нас уже исключалась вместе с переменной . Как вы поступите в таком случае? При таком положении нам нужно перейти к исключению переменной . Если же  уже исключались, тогда переходим к ,  и т. д.

Рассмотрим систему уравнений на таком этапе, когда уже исключилась переменная :

Такая система уравнений после преобразования выглядит так:

Вы наверное уже обратили внимание, что вместе с исключились и . Поэтому решение методом Гаусса продолжаем исключением переменной из всех уравнений системы, а начнём мы с третьего уравнения:

Чтобы завершить уравнение прямым ходом метода Гаусса, необходимо исключить последнюю неизвестную переменную из последнего уравнения:

Допусти, что система уравнений стала:

В этой системе нет ни одного уравнения, которое бы сводилось к . В данном случае можно было бы говорить о несовместности системы. Дальше непонятно, что же делать? Выход есть всегда. Для начала нужно выписать все неизвестные, которые стоят на первом месте в системе:

В нашем примере это , и . В левой части системы оставим только неизвестные, которые выделены зелёным квадратом а в правую перенесём известные числа, но с противоположным знаком. Посмотрите на примере, как это выглядит:

Можно придать неизвестным переменным с правой части уравнений свободные (произвольные) значения: , , , где , ,  – произвольные числа.

Теперь в правых частях уравнений нашей системы имеются числа и можно приступать к обратному ходу решения методом Гаусса.

В последнем уравнении системы получилось: , и теперь мы легко найдём решение в предпоследнем уравнении: , а из первого уравнения получаем:

= =

В итоге, получился результат, который можно и записать.

Ответ

,

,

,

,

,

.

Примеры решения методом Гаусса

Выше мы подробно расписали решение системы методом Гаусса. Чтобы закрепить материал, решим несколько примеров, в которых опять нам поможет метод Гаусса. Соответственно, начнём с самой простой системы.

Пример 1

Задача 

Решить систему линейных алгебраических уравнений методом Гаусса:

Решение

Выписываем матрицу, куда добавляем столбец свободных членов:

Прежде всего мы смотрим на элемент, который находится в матрице в левом верхнем углу (первая строка, первый столбец). Для наглядности выделим цифру зелёным квадратом. На этом месте практически всегда стоит единица:

Так как мы должны использовать подходящее элементарное преобразование строк и сделать так, чтобы элемент, который находится в матрице под выделенной цифрой превратился в . Для этого можно ко второй строке прибавить первую строку и умножить на .Однако, не сильно хочется работать с дробями, поэтому давайте постараемся этого избежать. Для этого нужно вторую строку умножить на (разрешающий элемент данного шага).

Соответственно, первая строка остаётся неизменной, а вторая поменяется:

Подбираем такое элементарное преобразование строк, чтобы во второй строке в первом столбце образовался . Для этого первую строку нужно умножить на и только после этого ко второй строке прибавить изменённую после умножения на вторую строку. Вот что получилось:

. Теперь прибавляем со второй строки первую строку . У нас получился , который записываем во вторую строку в первый столбец. Также решаем и остальные элементы матрицы. Вот что у нас получилось:

Как всегда у нас первая строка осталась без изменений, а вторая с новыми числами.

Итак, у нас получился ступенчатый вид матрицы:

Записываем новую систему уравнений:

Для проверки решаем систему обратным ходом. Для этого находим сначала :

Так как найден, находим :

.

Подставляем в изначальную нашу систему уравнений найденные и :

и .

Как видите из решения, система уравнений решена верно. Запишем ответ.

Ответ

Выше мы решали систему уравнений в двумя неизвестными, а теперь рассмотрим систему уравнений с тремя неизвестными.

Пример 2

Задача

Решить систему уравнений методом Гаусса:

Решение

Составляем матрицу, куда вписываем и свободные члены:

Что нам надо? Чтобы вместо цифры 2 появился 0. Для этого подбираем ближайшее число. Например, можно взять цифру -2 и на неё перемножить все элементы первой строки. Значит, умножаем , а потом прибавляем, при этом задействуем вторую строку: . В итоге у нас получился нуль, который записываем во вторую строку в первый столбец. Затем , и . Аналогично, и . И умножаем свободный член . Так и запишем следующую матрицу. Не забывайте, что первая строка остаётся без изменений:

Дальше необходимо проделать те же самые действия по отношению к третьей строке. То есть, первую строку нужно умножать не на (-2), а на цифру 3, так как и в третьей строке нужно коэффициенты привести у нулю. Также первую строку умножаем на 3 и прибавляем третью строку. Получается так:

Теперь нужно обнулить элемент 7, который стоит в третьей строке во втором столбце. Для этого выбираем цифру (-7) и проделываем те же действия. Однако, необходимо задействовать вторую строку. То есть, вторую строку умножаем на (-7) и прибавляем с третьей строкой. Итак, . Записываем результат в третью строку. Такие же действия проделываем и с остальными элементами. Получается новая матрица:

В результате получилась ступенчатая система уравнений:

Сначала находим : ,

.

Обратный ход:

Итак, уравнение системы решено верно.

Ответ

,

,

.

Пример 3

Система с четырьмя неизвестными более сложная, так как в ней легко запутаться. Попробуем решить такую систему уравнений.

Задача

Решите систему уравнений методом Гаусса:

Решение                                                                

В уравнении , то есть – ведущий член и пусть  ≠ 0

Из данного уравнения составим расширенную матрицу:

Теперь нужно умножить последние три строки (вторую, третью и четвёртую) на: , , . Затем прибавим полученный результат ко второй, третьей и четвёртой строкам исключаем переменную из каждой строки, начиная не с первой, а не со второй. Посмотрите, как изменилась наша новая матрица и в теперь стоит 0.

Поменяем вторую и третью строку местами и получим:

Получилось так, что = b и тогда, умножая вторую строку на (-7/4) и результат данной строки, прибавляя к четвёртой, можно исключить переменную из третьей и четвёртой строк:

Получилась такая матрица:

Также, учитывая, что  = , умножим третью строку на: 13,5/8 = 27/16, и, полученный результат прибавим к четвёртой, чтобы исключить переменную и получаем новую систему уравнений:

Теперь необходимо решить уравнение обратным ходом и найдём из последнего, четвёртого уравнения ,

из третьего: = = =

второе уравнение находим: = = = 2,

из первого уравнения: = .

Значит, решение системы такое: (1, 2, -1, -2).

Ответ

,

,

,

.

Добавим ещё несколько примеров для закрепления материла, но без такого подробного описания, как предыдущие системы уравнений.

Пример 4

Задача

Решить систему уравнений методом Гаусса:

Решение

Записываем расширенную матрицу системы:

Сначала смотрим на левое верхнее число:

Как выше уже было сказано, на этом месте должна стоять единица, но не обязательно. Производим такие действия: первую строку умножаем на -3, а потом ко второй строке прибавляем первую:

 

Производим следующие действия: первую строку умножаем на -1. Затем к третьей строки прибавляем вторую:

Теперь вторую строку умножаем на 1, а затем к третьей строке прибавляем вторую:

Получился ступенчатый вид уравнения:

Проверяем:

,

,

,

,

.

.

  Ответ

,

,

.

Заключение

Итак, вы видите, что метод Гаусса – интересный и простой способ решения систем линейных алгебраических уравнений. Путём элементарных преобразований нужно из системы исключать неизвестные переменные, чтобы систему превратить в ступенчатый вид. Данный метод удобен тем, что всегда можно проверить, правильно ли решено уравнение. Нужно просто подставить найденные неизвестные в изначальную систему уравнений.

Если элементы определителя не равняются нулю, тогда лучше обратиться к методу Крамера, а если же элементы нулевые, тогда такие системы очень удобно решать благодаря методу Гаусса.

Предлагаем ещё почитать учебники, в которых также описаны решения систем методом Гаусса.

Литература для общего развития:

Умнов А. Е. Аналитическая геометрия и линейная алгебра, изд. 3: учеб. пособие – М. МФТИ – 2011 – 259 с.

Карчевский Е. М. Лекции по линейной алгебре и аналитической геометрии, учеб. пособие – Казанский университет – 2012 – 302 с.

Системы линейных уравнений. Метод Гаусса

Рассмотрим систему линейных уравнений:

   

С этой системой связываются две матрицы: матрица коэффициентов

   

и расширенная матрица — с присоединенными свободными членами:

   

Элементарными преобразованиями системы линейных уравнений называются:

1. умножение уравнения на отличное от нуля число;

2. прибавление к одному уравнению любого другого, умноженного на любое число;

3. перестановка уравнений местами.

Теорема. Любая система линейных уравнений с помощью элементарных преобразований и, может быть, изменением нумерации неизвестных, может быть приведена к системе с трапециевидной матрицей.

Доказательство. Проводим элементарные преобразования только над строками матрицы , как в доказательстве теоремы о ранге матрицы. Возможно, при этом придется изменить нумерацию неизвестных. Приводим систему уравнений к виду

   

Если хотя бы одно из чисел отлично от нуля, то данная система уравнений решений не имеет (несовместна). Если же все они равны нулю, то последние равенств не несут никакой информации и могут быть отброшены. Тогда, если , то неизвестным можно придавать произвольные значения, а неизвестные находим из решения системы с треугольной матрицей

   

Эту систему удобно решать, определив из -го уравнения , затем из -го и т.д. Таким образом, можно выразить переменные через и получить общее решение системы. Если , то система (в случае совместности) имеет единственное решение.

Преобразование системы уравнений к системе с трапециевидной матрицей называется прямым ходом метода Гаусса. Последовательное вычисление неизвестных в порядке называется обратным ходом.

Пример. Решить систему линейных уравнений

   

Решение. Составим расширенную матрицу системы:

   

Первую строку умножим на 3 и вычтем из второй. Затем первую строку умножим на 2 и вычтем из третьей. Получим

   

Далее вторую строку прибавим к третьей и отбросим нулевую строку, получим

   

Запишем полученные уравнения:

   

Из второго уравнения выразим :

   

Полученное выражение подставляем в первое уравнение и выражаем из него :

   

Ответ. Общее решение данной системы:

   

Задачи.

1. Решите систему линейных уравнений

   

2. Решите систему линейных уравнений

   

3. Решите систему линейных уравнений

   

Численные методы: решение систем линейных уравнений

В прикладных задачах часто возникает необходимость решать системы линейных уравнений.

Система линейных алгебраических уравнений с n неизвестными  —  это система уравнений вида

                                     (1)

Слово система означает, что все уравнения рассматриваются как одно целое.

В общем случае у нас имеется m — уравнений, n — количество неизвестных. x1x2,…, xn — неизвестные, которые следует определить.

В системе (1)  – фиксированные коэффициенты,  b1b2, …, bm — свободные члены — предполагаются известными.

Система (1) называется однородной, если все её свободные члены равны нулю (b1 = b2 = … = bm = 0), иначе — неоднородной.

Система (1) называется квадратной, если число m уравнений равно числу n неизвестных.

Задача состоит в том, чтобы найти такие  которые удовлетворяют всем уравнениям (1).

В частном случае мы имеем одно линейное уравнение:

Конечно, такое уравнение легко решить, если предположить, что коэффициент  не равен 0, имеем:  = .

Очевидно, в общем случае имеются 3 варианта решений: система имеет ни одного решения, имеет одно решение, более одного решения.

Система (1) называется совместной, если она имеет хотя бы одно решение, и несовместной, если нет ни одного решения.

Система линейных уравнений может быть представлена в матричной форме как:

или:

Ax = b

Здесь A — это матрица системы, x — столбец неизвестных, а b — столбец свободных членов.

Если к матрице A приписать справа столбец свободных членов, то получившаяся матрица называется расширенной.

Рассмотрим, например, систему вида и поймем, как найти ее решение:

                                      (2)

Предположим на минуту, что в первом уравнении y отсутствует, а во втором отсутствует x, тогда мы имели бы решение именно то решение, которое нам нужно.

Вопрос: как исходную систему привести к такому виду и можно ли это сделать.

Заметим, что с тождествами мы можем делать следующие вещи: домножать на одно и то же число, отличное от 0, складывать, вычитать и тд, это похоже с тем, что вы раскладываете монеты по своим карманам, не меняя общей суммы.

От этих операций тождество не меняется.

В системе (2) у нас два тождества, домножим второе тождество на 2 и вычтем из первого, получим:

                                      (3)

Формально у нас есть еще старое тождество , но оно нам не понадобится (подумайте, почему).

Система (3) точно такая же, как система (2).

Из второго уравнения системы (3) сразу получим:

 

Никто не мешает нам подставить это значение в первое уравнение:

Отсюда сразу находим, что

Итак, путем простых действий мы нашли, что система (2) может быть представлена в виде:

Именно такие естественные соображения приводят к общему методу решения систем линейных уравнений, известному как метод исключения или метод Гаусса.

Метод Гаусса является одним из самых распространенных прямых методов решения систем линейных уравнений Ax = b:

Опишем этот метод в общем случае.

Вначале исходная система приводится к верхнетреугольному виду.

Это достигается следующей последовательностью преобразований (прямой ход).

Будем считать для удобства, что элемент aij исходной матрицы и компоненты вектора bi есть, соответственно, элементы aij (1) первого шага преобразованной матрицы A1 и преобразованного вектора b1:A = A1, b=b1

Далее, на втором шаге прибавим к второй строке первую, умноженную на  

Аналогично поступим со всеми оставшимися строками, т.е. прибавим к каждой i-ой строке i=2,3,…,N, первую, умноженную на коэффициент  

При этом соответственно изменится и вектор b1.  

Таким образом, 2 шаг.

Имеем систему уравнений A2x = b2:

где

3 шаг.

Прибавим к новой третьей строке новую вторую, умноженную на  

То же самое сделаем с остальными строками 4,5,…,N, т.е. прибавим к i-ой строке вторую, умноженную на  

При этом получим систему A3x = b3:

(k+1)-ый шаг:

Здесь

Поступая так и далее, на шаге N-1 получаем верхнетреугольную систему:

При этом, мы также получили матрицу C переводных коэффициентов, имеющую вид:

Решение полученной треугольной системы  как легко видеть, имеет вид (обратный ход метода Гаусса):

Заметим, что при прямом ходе метода Гаусса может возникнуть ситуация, когда происходит деление на нуль, да и вообще, желательно не делить на малое число, чтобы не накапливалась ошибка.

Поэтому метод Гаусса обычно проводят с частичным выбором главного элемента, то есть после каждого шага (пусть это был k-й шаг) переставляют строки с номерами k,k+1,…,N таким образом, чтобы на месте kk оказался элемент  наибольший из всех в k-ом столбце при m>k (при этом, естественно, переставляются и компоненты вектора b).

Можно для максимальной точности переставлять также и столбцы преобразуемой матрицы, чтобы на месте kk оказался максимальный элемент из всех с индексами больше, либо равными k.

Эта процедура называется методом Гаусса с выбором главного элемента. Она несколько повышает точность по сравнению с частичным выбором главного элемента, но весьма неудобна, в том числе для программирования, поскольку при перестановке строк компоненты искомого вектора x переставлять не надо, тогда как при перестановке столбцов надо переставлять и соответствующие компоненты вектора x.

Опишем обратный ход метода Гаусса в несколько иной форме (треугольное разложение).

Введем матрицы Mk по правилу:

На каждом шаге метода Гаусса получается некоторая промежуточная матрица: 

 и вектор  

Нетрудно видеть, что

Вопрос. Почему

Если производить также выбор главных элементов, то необходимо использовать оператор P перестановки индексов l и m, матричные элементы которого равны:

При применении оператора перестановки индексов к матрице слева, меняются местами строки матрицы и компоненты свободного вектора (PAx = Pb), если же его применить справа к матрице, то меняются местами ее столбцы и компоненты решения

Существует большой класс так называемых итерационных методов решения систем уравнений, аналогичных итерационным методам нахождения корней нелинейных уравнений.

Итерационные методы последовательно уточняют решение, отправляясь от начального приближения.

При выполнении условий сходимости они позволяют достичь любой точности просто повторением итераций.

Преимущество этих методов в том, что часто они позволяют достичь решения с заранее заданной точностью быстрее, а также позволяют решать большие системы уравнений.

Идея состоит в том, чтобы найти неподвижную точку матричного уравнения

                                     (5)

эквивалентного начальной системе линейных алгебраических уравнений.

При итерации  в правой части уравнения заменяется, например, в методе Якоби (метод простой итерации) приближение, найденное на предыдущем шаге:

.

Термин неподвижная точка становится ясен, если вы внимательно посмотрите на уравнение (5), по самому своему смыслу величина Х является неподвижной точкой.

Более подробное описание методов решения систем линейных уравнений можно найти в специальной литературе, наша задача дать обзор методов и основные идеи решения такого рода задач.

Обусловленность линейных систем, погрешность

При решении абстрактной задачи Ax = b, где A — оператор произвольной природы, важным моментом является корректность ее постановки.

Задача считается корректной, если решение существует и единственно и , кроме того, решение непрерывно зависит от данных (то есть, при  также стремится к нулю).

Однако и непрерывная зависимость от входных данных может иметь свои нюансы.

Чем меньшее (большее) изменение решения вызывает вариация входных данных, тем более хорошо (плохо) обусловленной считается задача.

Понятие обусловленности является тем более существенным для численных методов, поскольку на практике входные данные известны, как правило, с некоторой погрешностью.

Кроме того, существуют ошибки округления, возникающие при вычислениях.

Таким образом, формально корректная задача, являясь плохо обусловленной, может оказаться разрешимой столь неточно, что в этом будет отсутствовать практический смысл.

Чем можно охарактеризовать количественно обусловленность для линейных систем?

Пусть A — квадратная NxN — матрица.

Рассмотрим задачу Ax = b.

Пусть также  некоторая норма в пространстве RN 

Норма оператора A определяется стандартно:

Обозначим y = Ax и введем число m по правилу:

Величина  называется числом обусловленности.

Очевидно:

  1.      
  2. если A — диагональная, то  (Для какой нормы, или для всех вышеприведенных?). Чем меньше число обусловленности C(A), тем лучше обусловлена система. Действительно, пусть  вариация правой части, а соответствующее изменение решения.

Тогда справедливо следующее неравенство:

 

Доказательство. Имеем:

Так как

то    

Аналогично, поскольку  

Объединяя два неравенства, окончательно получаем для оценки погрешности:

 

В начало

Содержание портала

Решение систем линейных уравнений методом Гаусса

1. Решение систем линейных уравнений методом Гаусса

2. Метод Гаусса – это метод последовательного исключения переменных

• Систему уравнений приводят к
эквивалентной ей системе с
треугольной матрицей. Это называется
прямым ходом.
• Из полученной треугольной системы
переменные находят с помощью
последовательных подстановок. Это
называется обратным ходом.

3. При выполнении прямого хода используют следующие преобразования:

1. Умножение или деление коэффициентов
свободных членов на одно и то же число;
2. Сложение и вычитание уравнений;
3. Перестановка уравнений системы;
4. Исключение из системы уравнений, в
которых все коэффициенты при
неизвестных и свободные члены равны
нулю.

4. Решить систему уравнений методом Гаусса

x y 5
2 x y 7
Нужно записать расширенную матрицу системы
1 1 5
2 1 7
Вертикальная черта внутри матрицы не несёт
никакого математического смысла – это
просто отчеркивание для удобства
оформления.
Матрица системы – это матрица,
составленная только из
коэффициентов при неизвестных.
Расширенная матрица системы – это
та же матрица системы плюс
столбец свободных членов, в
данном случае.

6. Решение. Умножим первую строку на (-2)

1 1 5
2 1 7
2 2 10
2 1 7

7. ко второй строке прибавим первую строку умноженную на -2

1 1 5
2 1 7
2 2 10
0 3 3
2 2 10
2 1 7

8. Разделим опять первую строку на (-2)

1 1 5
2 1 7
2 2 10
0 3 3
2 2 10
2 1 7
1 1 5
0 3 3
строка, которую ПРИБАВЛЯЛИ – не изменилась.
Всегда меняется строка, К КОТОРОЙ ПРИБАВЛЯЮТ.

9. Цель элементарных преобразований –

Цель элементарных преобразований

привести матрицу к ступенчатому виду.
Сам термин «ступенчатый вид» не
вполне теоретический, в научной и
учебной литературе он часто
называется трапециевидный
вид или треугольный

10. В результате элементарных преобразований получена эквивалентная исходной система уравнений

В результате элементарных преобразований
получена эквивалентная исходной система уравнений
x y 5
2 x y 7
x y 5
y 1
Выполняем обратный ход, т.е. подстановку в первое
уравнение вместо у,
х =-5+у
х=-5+1
х=-4
Ответ: (-4; 1)

11. Решить систему уравнений методом Гаусса

3 x 2 y z 4
2 x y 3z 9
x 2 y 2z 3
Решение.
Переставим третье уравнение на место первого и запишем расширенную
матрицу:
x 2 y 2z 3
3 x 2 y z 4
2 x y 3z 9
1 2 2 3
3 2 1 4
2 1 3 9

12.

Чтобы в первом столбце получить а2=а3=0, умножим 1-ю строку сначала на 3, а затем на 2 и вычтем результаты из 2-й и 3-й строк 1 2 2 3
3 2 1 4
2 1 3 9
1 2 2 3
0 8 7 5
0 3 1 3

13. Разделим 2-ю строку на 8, полученные результаты умножим на 3 и вычтем из 3-й строки

1 2 2 3
3 2 1 4
2 1 3 9
1 2 2 3
0 1 7 5
8 8
0 3 1 3
1 2 2 3
0 8 7 5
0 3 1 3
1 2 2 3
0 3 21 15
8
8
0 3 1 3
1 2 2
3
21
15
0
3
8
8
39
0 0 13
8
8

14. Запишем новую эквивалентную систему с учетом расширенной матрицы

x 2 y 2z 3
7
5
y z
8
8
13
39
z
8
8
x 2 y 2z 3
7
5
y z
8
8
13
39
z
8
8
Выполняем обратный ход, с помощью
последовательных подстановок находим
неизвестные
13
39
z
z 3
8
8
7
5
5 21 16
y 3
y
2
8
8
8 8
8
x 2 2 2 3 3 x 3 4 6 1
Ответ: (1; 2; 3)

в чем суть, решение системы уравнений, примеры с объяснением

Благодаря великим ученым было открыто множество эффективных теорем для работы со сложными математическими задачами. Один из таких примеров — метод Гаусса.

Метод Гаусса — что это такое

Метод Гаусса представляет собой методику эквивалентного преобразования исходной системы линейных уравнений в систему, решаемую существенно проще, чем исходный вариант.

Метод Гаусса используют для решения систем линейных алгебраических формул. Такой способ обладает рядом важных преимуществ:

  1. Нет необходимости сравнивать уравнения для оценки совместимости.
  2. Решение систем равенств, в которых число определителей совпадает или не совпадает с количеством неизвестных переменных.
  3. Поиск решений для уравнений с нулевым определителем.
  4. Сравнительно небольшое количество вычислительных операций для получения результата.

Основные определения и обозначения

Матрицы: определение и свойства

Такие системы являются наиболее удобным способом представления данных, с которыми впоследствии производят манипуляции. Матрица имеет вид прямоугольника для удобства расчетов. При использовании метода Гаусса работа осуществляется с треугольными матрицами, при записи которых применяется прямоугольник с нулями на тех местах, где числа отсутствуют. Часто нули не записывают, а только подразумевают.

Важным параметром матрицы является размер:

  • ширина — это количество строк, обозначают буквой m;
  • длину выражают числом столбцов, записывают буквой n.
Источник: bigpicture.ru

Размер матрицы будет записан в формате А m*n. В случае, когда m=n, матрица является квадратной, а m=n служит ее порядком. Номера строк и столбцов изменяются.

Определитель

Матрица обладает крайне важной характеристикой. Таким параметром является определитель. Данную величину рассчитывают с помощью диагонали. Для этого в матрице необходимо провести воображаемые диагональные линии. Затем следует найти произведение элементов, которые располагаются на этих диагоналях, а полученные значения суммировать таким образом:

  1. Если диагональ обладает наклоном в правую сторону, то знак «+».
  2. Для диагоналей, наклоненных влево, знак «–».
Источник: wp.com

Рассчитать определитель представляется возможным лишь в случае работы с квадратной матрицей.

Если необходимо определить данный параметр для прямоугольной матрицы, то следует выполнить следующие манипуляции:

  • из числа строк и числа столбцов выбрать наименьшее и обозначить его k;
  • отметить в матрице произвольным образом k столбцов и k строк.

Элементы, которые расположены на пересечении отмеченных столбцов и строк, образуют новую квадратную матрицу. В случае, когда определитель является числом, не равным нулю, то данный параметр будет обозначен как базисный минор первоначальной прямоугольной матрицы. Перед решением систем уравнений методом Гаусса полезно рассчитать определитель. Если данная характеристика равна нулю, то матрица имеет бесконечное множество решений либо не имеет их вовсе. В таком случае потребуется определить ранг матрицы.

Классификация систем

Ранг матрицы является распространенным понятием. Он обозначает максимальный порядок ее определителя, который не равен нулю. По-другому можно сказать, что ранг матрицы представляет собой порядок базисного минора. Исходя из данного критерия, СЛАУ классифицируют на несколько типов. В совместных системах, которые состоят лишь из коэффициентов, ранг основной матрицы совпадает с рангом расширенной. Для подобных систем характерно одно или множество решений. По этой причине совместные системы подразделяют на следующие типы:

  • определенные, обладающие одним решением, в которых наблюдается равенство ранга матрицы и количество неизвестных;
  • неопределенные;
  • обладающие бесконечным числом решений с рангом матрицы, который меньше количества неизвестных.

В несовместных системах ранги, характеризующие основную и расширенную матрицы, отличаются. С помощью метода Гаусса в процессе решения можно прийти либо к однозначному доказательству несовместности системы, либо к решению общего вида для системы, обладающей бесконечным количеством решений.

Источник: asiaplustj.info

Основные правила и разрешаемые преобразования при использовании метода Гаусса

Перед тем, как решать систему, необходимо ее упростить. На данном этапе выполняют элементарные преобразования, которые не влияют на конечный результат. Определенные манипуляции справедливы лишь в случае матриц, исходниками которых являются СЛАУ. Список элементарных преобразований:

  1. Перестановка строк. При перемене записей в системе местами ее решение не меняется. Можно менять место строк в матрице, учитывая столбец со свободными членами.
  2. Произведение всех элементов строк и некоторого коэффициента. Сокращаются большие числа в матрице, и исключаются нули. При этом множество решений сохраняется без изменений, а дальнейшие манипуляции существенно упрощаются. Важным условием является отличие от нуля коэффициента.
  3. Удаление строк, которые содержат пропорциональные коэффициенты. Данное преобразование следует из предыдущего пункта. При условии, что две или более строк в матрице обладают пропорциональными коэффициентами, то при произведении или делении одной из строк на коэффициент пропорциональности получают две или более абсолютно одинаковые строки. В этом случае лишние строки исключают, оставляя только одну.
  4. Удаление нулевой строки. Бывают случаи, когда в процессе манипуляций с уравнениями возникает строка, все элементы которой, в том числе свободный член, равны нулю. Нулевую строку допустимо исключать из матрицы.
  5. Суммирование элементов одной строки с элементами другой, умноженными на некоторый коэффициент, в соответствующих столбцах. Данное преобразование имеет наиболее важное значение из всех перечисленных.

Особенности использования метода Гаусса для решения СЛАУ

На первом этапе система уравнений записывается в определенном виде. Пример выглядит следующим образом:

Источник: wp.com

Коэффициенты необходимо представить в виде таблицы. С правой стороны в отдельном столбце записаны свободные члены. Данный блок отделен для удобства решения. Матрицу со столбцом со свободными членами называют расширенной.

Источник: wp.com

Затем основная матрица с коэффициентами приводится к верхней треугольной форме. Данное действие является ключевым моментом при решении системы уравнений с помощью метода Гаусса. По итогам преобразований матрица должна приобрести такой вид, чтобы слева внизу находились одни нули:

Источник: wp.com

При записи новой матрицы в виде системы уравнений можно отметить, что последняя строка уже содержит значение одного из корней, которое в дальнейшем подставляется в уравнение выше для нахождения следующего корня и так далее. Подобное описание позволяет разобраться в методе Гаусса в общих чертах.

Обратный и прямой ход метода Гаусса

В первом случае необходимо представить запись расширенной матрицы системы. При выполнении обратного метода Гаусса далее в главную матрицу добавляют столбец со свободными членами.

Источник: wp.com

Суть такого способа заключается в выполнении элементарных преобразований, по итогам которых данная матрица приводится к ступенчатому или треугольному виду. В этом случае над или под главной диагональю матрицы располагаются только нули.

Источник: wp.com

Варианты дальнейших действий:

  • перемена строк матрицы местами, при наличии одинаковых или пропорциональных строк их можно исключить, кроме одной;
  • деление либо умножение строки на любое число, не равное нулю;
  • удаление нулевых строк;
  • добавление строки, умноженной на число, не равное нулю, к другой строке.

Имея преобразованную систему с одной неизвестной Xn, которая становится известной, можно выполнить поиск в обратном порядке остальных неизвестных с помощью подстановки известных х в уравнения системы, вплоть до первого. Данный способ называют обратным методом Гаусса.

Примеры решений с объяснением

Пример 1

Требуется решить с помощью метода Гаусса систему линейных уравнений, которая выглядит следующим образом:

Источник: wp. com

Решение

Необходимо записать расширенную матрицу:

Источник: wp.com

Затем нужно выполнить преобразования. В результате матрица должна приобрести треугольный вид. Для этого следует умножить первую строку на (3) и умножить вторую строку на (-1). В результате суммирования второй и первой строк получается следующее:

Источник: wp.com

Далее следует умножить третью строку на (-1). После добавления третьей строки ко второй получаем следующие преобразования:

Источник: wp.com

После этого необходимо умножить первую строку на (6) и вторую строку на (13). Далее следует добавить вторую строку к первой:

Источник: wp.com

После того, как система преобразована, остается вычислить неизвестные:

\(x_{3}=\frac{98}{49}=2\)

\(x_{2}=\frac{14-7x_{3}}{6}=\frac{14-7*2}{6}=0\)

\(x_{3}=\frac{-9+5x_{2}+6x_{3}}{3}=\frac{-9+5*0+6*2}{3}=1\)

Данный пример демонстрирует единственное решение системы.

Источник: supertics. com

Пример 2

Необходимо решить систему уравнений, которая выглядит следующим образом:

Источник: wp.com

Решение

Необходимо составить матрицу:

Источник: wp.com

Согласно методу Гаусса уравнение первой строки по итогам преобразований не меняется. Удобнее, когда левый верхний элемент матрицы обладает наименьшим значением. В таком случае первые элементы остальных строк после преобразований будут равны нулю. Таким образом, составленная матрица будет решаться проще, если на место первой строки поставить вторую:

вторая строка:

\(k = (-a_{21} /a_{11}) = (-3/1) = -3\)

\(a»_{21} = a_{21} + k×a_{11} = 3 + (-3)×1 = 0\)

\(a» _{22} = a_{22} + k×a _{12} = -1 + (-3)×2 = -7\)

\(a»_{ 23} = a_{23} + k×a_{13} = 1 + (-3)×4 = -11\)

b» 2 = b 2 + k×b 1 = 12 + (-3)×12 = -24

третья строка: 

\(k = (-a_{31} /a_{11}) = (-5/1) = -5\)

\(a»_{31} = a_{31} + k×a_{11} = 5 + (-5)×1 = 0\)

\(a»_{32} = a_{32} + k×a_{12} = 1 + (-5)×2 = -9\)

\( a»_{33} = a_{33} + k×a_{13} = 2 + (-5)×4 = -18\)

\( b»_3 = b_3 + k×b_1 = 3 + (-5)×12 = -57\)

Матрица с промежуточными результатами манипуляций будет иметь следующий вид:

Источник: wp. com

Благодаря некоторым операциям можно придать матрице наиболее удобный вид. К примеру, вторую строку можно избавить от всех «минусов» путем умножения каждого элемента на «-1». Можно заметить, что для третьей строки характерны все элементы, кратные трем. В этом случае строка сокращается с помощью произведения каждого элемента на «-1/3». Минус позволит удалить отрицательные значения.

Источник: wp.com

Далее следует приступить к манипуляциям со второй и третьей строками. Необходимо суммировать третью и вторую строки. Вторая строка при этом умножается на такой коэффициент, при котором элемент а 32 будет равен нулю.

\(k = (-a_{32} /a_{22}) = (-3/7) = -3/7\)

В случае, когда некоторые преобразования приводят в результате к получению не целого числа, следует оставить его в этом виде. Таким образом, вычисления будут более точными. Затем при получении ответов можно определиться с его дальнейшем округлением или переводом в другую форму записи.

\(a»_{32} = a_{32} + k×a_{22} = 3 + (-3/7)×7 = 3 + (-3) = 0\)

\(a»_{33} = a_{33} + k×a_{23} = 6 + (-3/7)×11 = -9/7\)

\(b»_3 = b_3 + k×b_2 = 19 + (-3/7)×24 = -61/7\)

Преобразованная матрица будет иметь следующий вид:

 

 

Матрица обладает ступенчатым видом. Дальнейшие преобразования с помощью метода Гаусса нецелесообразны. В этом случае можно удалить из третьей строки общий коэффициент «-1/7».

Источник: wp.com

Затем необходимо представить запись матрицы в виде системы уравнений для вычисления корней.

x + 2y + 4z = 12 (1)

7y + 11z = 24 (2)

Найти корни можно обратным методом Гаусса. Уравнение (3) содержит значение z:

y = (24 — 11×(61/9))/7 = -65/9

С помощью первого уравнения можно определить х:

x = (12 — 4z — 2y)/1 = 12 — 4×(61/9) — 2×(-65/9) = -6/9 = -2/3

Подобная система является совместной и определенной, для которого характерно единственное решение. Ответ будет следующим:

x 1 = -2/3, y = -65/9, z = 61/9.

Метод Гаусса предполагает последовательное исключение неизвестных. Методика справедлива в случае решения квадратных систем линейных алгебраических уравнений. Несмотря на простоту метода, многие студенты сталкиваются с некоторыми трудностями в процессе поиска правильного решения. Это связано с наличием знаков «+» и «-». Поэтому для решения СЛАУ требуется проявить внимательность. А получить квалифицированную помощь можно на ресурсе Феникс.Хелп.

Решение систем линейных уравнений методом Гаусса

Теория

Классическим методом решения систем линейных алгебраических уравнений является метод Гаусса (метод исключений Гаусса). Суть метода — это последовательное исключение неизвестных, т.е. когда с помощью элементарных преобразований система уравнений приводится к равносильной системе ступенчатого вида, из которой последовательно, начиная с последних переменных, находятся все остальные переменные.

Матрица, составленная из все ai,j, называется основной матрицей системы. Если к этой матрице добавить вектор столбец, составленный из bi, то такая матрица называется расширенной матрицей системы.

Теорема Кронекера-Капелли (условие совместности системы): системат совместна тогда и только тогда, ранг ее основной матрицы равен рангу ее расширенной матрицы.

Алгоритм решения СЛАУ методом Гаусса подразделяется на два этапа:

  • На первом этапе (прямой ход) система приводится ступенчатой или треугольной форме. Вычтем из второго уравнения системы первое, умноженное на такое число, чтобы обнулился коэффициент при x1. Затем таким же образом вычтем первое уравнение из третьего, четвертого и т.д. Тогда исключаются все коэффициенты первого столбца, лежащие ниже главной диагонали. Затем при помощи второго уравнения исключим из третьего, четвертого и т.д. уравнений коэффициенты второго столбца. Последовательно продолжая этот процесс, исключим из матрицы все коэффициенты, лежащие ниже главной даигонали.
  • На втором этапе (обратный ход) выражаем все получившиеся базисные переменные через небазисные и построим фундаментальную систему решений. Если все переменные являются базисными, то получим единственное решение системы линейных уравнений. Эта процедура начинается с последнего уравнения, из которого выражают соответствующую базисную переменную (а она там всего одна) и подставляют в предыдущие уравнения, и так далее, поднимаясь по «ступенькам» наверх. Каждой строчке соответствует ровно одна базисная переменная, поэтому на каждом шаге, кроме последнего (самого верхнего), ситуация в точности повторяет случай последней строки.

Системы линейных уравнений: исключение Гаусса

Решение линейной системы с матрицами с использованием исключения Гаусса

После нескольких уроков, в которых мы неоднократно упоминали, что мы охватываем основы, необходимые для последующего изучения того, как решать системы линейных уравнений, пришло время для нашего урока сосредоточиться на полной методологии, которой нужно следовать, чтобы найти решения. для таких систем.

Что такое гауссовское исключение

Исключение Гаусса — это название метода, который мы используем для выполнения трех типов операций со строками матрицы над расширенной матрицей, полученной из линейной системы уравнений, чтобы найти решения для такой системы. Этот метод также называется сокращением строк и состоит из двух этапов: прямого исключения и обратной замены.

Эти два шага метода исключения Гаусса различаются не операциями, которые вы можете использовать с их помощью, а результатом, который они производят. Шаг прямого исключения относится к сокращению строки, необходимому для упрощения рассматриваемой матрицы до ее эшелонированной формы. Такой этап имеет целью продемонстрировать, имеет ли система уравнений, изображенная в матрице, единственное возможное решение, бесконечное множество решений или просто отсутствие решения.Если обнаружено, что система не имеет решения, то нет причин продолжать сокращение строки матрицы на следующем этапе.

Если возможно получить решения для переменных, входящих в линейную систему, то выполняется этап исключения Гаусса с обратной подстановкой. На этом последнем шаге будет получена сокращенная форма матрицы, которая, в свою очередь, дает общее решение системы линейных уравнений.

Правила исключения Гаусса такие же, как правила для трех элементарных операций со строками, другими словами, вы можете алгебраически оперировать строками матрицы следующими тремя способами (или комбинацией):

  1. Перестановка двух рядов
  2. Умножение строки на константу (любую константу, отличную от нуля)
  3. Добавление строки к другой строке

Итак, решение линейной системы с матрицами с использованием исключения Гаусса оказывается структурированным, организованным и довольно эффективным методом.

Как выполнить исключение по Гауссу

На самом деле это не установленный набор шагов исключения Гаусса, которым нужно следовать, чтобы решить систему линейных уравнений, это все о матрице, которую вы имеете в руках, и необходимых операциях со строками для ее упрощения. Для этого давайте поработаем над нашим первым примером исключения Гаусса, чтобы вы могли начать изучать весь процесс и интуицию, которая необходима при работе с ними:

Пример 1

Обратите внимание, что в этот момент мы можем заметить, что эта система линейных уравнений разрешима с единственным решением для каждой из ее переменных.То, что мы выполнили до сих пор, — это первый этап сокращения ряда: прямое исключение. Мы можем продолжить упрощение этой матрицы еще больше (что приведет нас ко второму этапу обратной подстановки), но нам это действительно не нужно, поскольку на этом этапе система легко разрешима. Таким образом, мы смотрим на получившуюся систему, чтобы решить ее напрямую:

  • Уравнение 5: Полученная линейная система уравнений для решения

Из этого набора мы можем автоматически заметить, что значение переменной z равно: z = -2. Мы используем это знание, чтобы подставить его во вторые уравнения для решения относительно y, и подставить значения y и z в первые уравнения для решения относительно x:

В последний раздел этого урока добавлено больше задач исключения Гаусса. Обязательно проработайте их, чтобы практиковаться.

Разница между устранением гаусса и устранением гаусса иордана

Разница между методом исключения Гаусса и исключения Гаусса Жордана заключается в том, что один создает матрицу в форме эшелона строк, а другой — матрицу в форме уменьшенного эшелона строки.Матрица формы эшелона строк имеет верхнюю треугольную композицию, где любые нулевые строки находятся внизу, а ведущие члены находятся справа от ведущего члена из строки выше. Уменьшенная форма эшелона выходит за рамки еще большего упрощения (иногда даже достигая формы единичной матрицы).

Уравнение 8: Разница между формой эшелона и формой ряда эшелонов

История исключения Гаусса и его названия весьма интересны, вы будете удивлены, узнав, что название «Гауссовский» было присвоено этой методологии по ошибке в прошлом веке. В действительности было обнаружено, что алгоритм одновременного решения системы линейных уравнений с использованием матриц и редукции строк записан в той или иной форме в древних китайских текстах, которые датируются еще до нашей эры. Затем в конце 1600-х годов Исаак Ньютон провел по этому уроку, чтобы заполнить то, что он считал пробелом в книгах по алгебре. После того, как название «Гауссиан» было уже установлено в 1950-х годах, термин Гаусса-Иордана был принят, когда геодезист У. Джордан усовершенствовал эту технику, чтобы он мог использовать такие вычисления для обработки своих наблюдаемых данных топографической съемки.Если вы хотите продолжить чтение увлекательной истории математиков исключения Гаусса, не бойтесь щелкнуть ссылку и прочитать.

На самом деле нет никакой физической разницы между исключением Гаусса и исключением Гаусса Джордана, оба процесса следуют одному и тому же типу операций со строками и их комбинациям, их различие зависит от результатов, которые они производят. Многие математики и учителя во всем мире будут относиться к устранению Гаусса и исключению Гаусса Джордана как к методам создания матрицы эшелонированной формы по сравнению с методом создания матрицы сокращенной формы эшелона, но на самом деле они говорят о двух стадиях сокращения строк. мы объяснили это в самом первом разделе этого урока (прямое исключение и обратная подстановка), и поэтому вы просто применяете операции со строками, пока не упростите рассматриваемую матрицу.Если вы дойдете до формы эшелона, вы обычно можете решить с ней систему линейных уравнений (до сих пор это то, что называлось бы исключением Гаусса). Если вам нужно продолжить упрощение такой матрицы, чтобы напрямую получить общее решение для системы уравнений, над которой вы работаете, в этом случае вы просто продолжаете работать с матрицей по строкам, пока не упростите ее до сокращенной формы эшелона. (это будет то, что мы называем частью Гаусса-Жордана, и которую можно также рассматривать как поворотное исключение Гаусса).

Мы оставим подробное объяснение форм сокращения строк и эшелонирования для следующего урока, поскольку сейчас вам нужно знать, что, если у вас нет единичной матрицы в левой части расширенной матрицы, которую вы решаете (в этом случае вы не используете не нужно ничего делать для решения системы уравнений, относящейся к матрице), метод исключения Гаусса (регулярное сокращение строк) всегда будет использоваться для решения линейной системы уравнений, которая была записана в виде матрицы.

Примеры исключения Гаусса

В качестве последнего раздела давайте поработаем еще несколько упражнений по исключению Гаусса (сокращение строк), чтобы вы могли больше попрактиковаться в этой методологии.На протяжении многих будущих уроков этого курса линейной алгебры вы обнаружите, что сокращение строк является одним из самых важных инструментов при работе с матричными уравнениями. Поэтому убедитесь, что вы понимаете все этапы решения следующих проблем.

Пример 2

Пример 3

Мы знаем, что для этой системы мы получим расширенную матрицу с тремя строками (поскольку система содержит три уравнения) и тремя столбцами слева от вертикальной линии (поскольку есть три разных переменных). В этом случае мы перейдем непосредственно к сокращению строк, и поэтому первая матрица, которую вы увидите в этом процессе, — это та, которую вы получите, преобразовав систему линейных уравнений в расширенную матрицу.

  • Уравнение 15: Строка, уменьшающая расширенную матрицу

Обратите внимание, как мы можем сразу сказать, что переменная z равна нулю для этой системы, поскольку третья строка результирующей матрицы показывает уравнение -9z = 0 . Мы используем это знание и проверяем вторую строку матрицы, которая предоставит уравнение 2y — 6z = 0 , подставив в это уравнение значение z = 0 \, в результате получится y \, также равное нулю.Таким образом, мы наконец подставляем оба значения y и z \ в уравнение, которое получается из первой строки матрицы: x + 4y + 3z = 1 , поскольку и y , и z \ , равны нулю, то это дает нам x = 1 . Итак, окончательное решение этой системы уравнений выглядит следующим образом:

  • Уравнение 16: Окончательное решение системы уравнений

Пример 4

Из чего видно, что последняя строка дает уравнение: 6z = 3 и, следовательно, z = 1/2. Мы подставляем это в уравнения, полученные во второй и первой строках (в указанном порядке), чтобы вычислить значения переменных x и y:

Пример 5

  • Решите следующую линейную систему, используя метод исключения Гаусса: Уравнение 21: Система линейных уравнений с двумя переменными
  • Транскрипция линейной системы в виде расширенной матрицы и редукции строк: Уравнение 22: Строка, уменьшающая расширенную матрицу
  • Что автоматически говорит нам y = 8 .Итак, подставляя это значение в уравнение из первой строки, получаем: 4x — 5y = 4x — 5 (8) = 4x — 40 = -6 4x = 34 \, и поэтому значение x равно: x = 172 \ frac {\ small17} {\ small2} 217 . И окончательное решение этой системы уравнений:

    Уравнение 23: Окончательное решение системы уравнений

Пример 6

Чтобы завершить наш урок на сегодня, у нас есть рекомендация по ссылке, чтобы дополнить ваши исследования: Исключение Гаусса — статья, которая содержит некоторую дополнительную информацию о сокращении строк, включая введение в тему и еще несколько примеров. Как мы уже упоминали ранее, будьте готовы продолжать использовать сокращение строк почти на всем протяжении этого курса линейной алгебры, так что до встречи на следующем уроке!

Системы линейных уравнений: исключение Гаусса

Системы линейных уравнений:
Решение методом исключения Гаусса
(стр. 6 из 7)

Разделы: Определения, Решение по графику, Подстановка, Исключение / добавление, исключение по Гауссу.


Решение трех переменных, линейных систем с тремя уравнениями сложнее, по крайней мере, на начальном этапе, чем решение систем с двумя переменными, потому что требуемые вычисления более грязный. Вам нужно будет очень аккуратно работать, и вы должны планируйте использовать много бумаги для заметок. Метод решения этих систем является расширением метода сложения двух переменных, поэтому сделайте конечно ты знаешь это метод хорошо и можно использовать его последовательно правильно.

Хотя метод решения основан на добавлении / исключении, попытка выполнить фактическое добавление имеет тенденцию становится очень запутанным, поэтому существует систематизированный метод решения трех или более переменных системы. Этот метод называется «методом исключения Гаусса» (с уравнения заканчиваются так называемой «строковой формой»).

Начнем с простого, и работаем над более сложными примерами.

  • Решите следующие проблемы система уравнений.
  • Довольно легко увидеть как действовать в этом случае. Я просто обратно заменю z -значение из третьего уравнения во второе уравнение, решите результат для л , а затем вставьте z и y в первое уравнение и решите результат для x .

      10 л 3 (3) = 11
      10 y 9 = 11
      10 y = 20
      y = 2

      5 х + 4 (2) (3) = 0
      5 x + 8 3 = 0
      5 x + 5 = 0
      5 x = 5
      x = 1

    Тогда решение ( х , y , z ) = (1, 2, 3).

Причина, по которой эта система была Легко решить, чтобы система была «треугольной»; это относится к уравнениям, имеющим форму треугольника, из-за нижних уравнений содержащий только более поздние переменные.

Дело в том, что в этом формат, система проста в решении. А исключение по Гауссу — это метод, который мы будем использовать для преобразования систем в эту верхнетреугольную форму, используя операции со строками, которые мы изучили, когда применили метод сложения.

  • Решите следующие проблемы система уравнений с использованием исключения Гаусса.
  • Уравнение не решается для переменной, поэтому мне придется проделать умножение и сложение чтобы упростить эту систему. Чтобы отслеживать свою работу, напишу вниз на каждом шагу, когда я иду. Но я сделаю свои вычисления на бумаге для заметок. Вот как я это сделал:

    Первое, что нужно сделать состоит в том, чтобы избавиться от ведущих терминов x в два ряда. А пока я просто посмотрю, какие строки будут легко расчистить; Я могу поменять строки позже, чтобы перевести систему в «верхний треугольной «формы. Нет правила, которое гласит, что я должен использовать x — срок из первого ряда, и в этом случае, думаю, будет проще используйте термин x из третьей строки, так как его коэффициент просто «1». Так что я умножу третью строку на 3, и добавьте его в первую строку.Я делаю вычисления на бумаге для заметок:

    … а потом записываю результатов:

    (Когда решали системы с двумя переменными, мы могли умножить строку, переписав систему в сторону, а затем добавить. Для этого нет места в система с тремя переменными, поэтому нам и нужна бумажка.)

    Предупреждение: поскольку я не на самом деле ничего не делаю с третьей строкой, я скопировал ее без изменений, в новую матрицу уравнений. Я б / у третий ряд, но на самом деле я не менял Это. Не путайте «использование» с «изменением».

    Чтобы получить меньшие числа для коэффициентов умножу первую строку пополам:

    Теперь умножу третий ряд на 5 и добавьте это ко второму ряд. Работаю на бумаге для заметок:

    … а потом записываю результаты: Авторские права Элизабет Стапель 2003-2011 Все права защищены

    Я ничего не делал с первым рядом, поэтому я скопировал его без изменений. Я работал с третий ряд, но я работал только на вторая строка, поэтому вторая строка обновляется, а третья строка копируется более без изменений.

    Хорошо, теперь x — столбец удаляется, за исключением ведущего члена в третьей строке. Так что дальше Приходится работать на колонке y .

    Предупреждение: Начиная с третьего уравнение имеет член x , Я больше не могу использовать его ни в одном из двух других уравнений (или я отменить мой прогресс). Я могу работать с на уравнении, но не на с Это.

    Если я добавлю в два раза больше первого строки во вторую строку, это даст мне ведущую 1 во втором ряду.Я не буду избавились от ведущего y -термина во втором ряду, но я его переделал (не вмешиваясь дробями) в более простую форму. (Вы должны сохранить обратите внимание на такого рода упрощения.) Сначала я делаю царапину работа:

    … а потом записываю результатов:

Теперь могу использовать второй ряд, чтобы убрать y -term в первом ряду. Вторую строку умножу на 7 и добавить. Сначала я царапаю работа:

… а потом записываю результатов:

Я могу сказать что z сейчас, но, чтобы быть внимательным, я разделю первую строку на 43. Затем я переставляю ряды, чтобы они имели верхнетреугольную форму:

Теперь я могу начать процесс обратного решения:

Тогда решение ( х , y , z ) = ( 2, 3, 1 ) .

Примечание: нет ничего священного о шагах, которые я использовал при решении указанной выше системы; там ничего не было особенно о том, как я решил эту систему. Вы могли бы работать в другом упорядочивайте или упрощайте разные строки, и все равно получите правильный ответ. Эти системы достаточно сложны, поэтому вряд ли один правильный способ вычисления ответа. Так что не беспокойтесь о том, «как она знала, что делать дальше? », потому что здесь нет правила.я только делал все, что пришло мне в голову; Я делал то, что казалось самым простым или что-то еще пришла в голову первая. Не волнуйтесь, если бы вы использовали совершенно другой шаги. Пока каждый шаг на пути верен, вы придумаете тот же ответ.


В приведенном выше примере я мог пошли дальше в своих вычислениях и более тщательно проработали строковые операции, очищая все термины y кроме этого во втором ряду и во всех терминах z кроме того, что в первой строке.Это то, что процесс тогда выглядело так:

Так я могу просто прочитать от значений x , л , и z , и мне не нужно заморачиваться с обратной заменой. Это более полное метод решения называется «методом исключения Гаусса-Жордана» (с уравнения, попадающие в так называемый «пониженный ряд-эшелон» форма»).Многие тексты доходят до исключения Гаусса, но я всегда было легче продолжать и делать Гаусс-Джордан.

Обратите внимание, что я выполнил две строковые операции сразу на этом последнем шаге перед переключением строк. Пока я не работа с и работа с в той же строке на том же шаге, это хорошо. В этом случае я работал с первой строкой и рабочий по второй и третий ряды.

<< Предыдущая Вверх | 1 | 2 | 3 | 4 | 5 | 6 | 7 | Вернуться к указателю Далее >>

Цитируйте эту статью как:

Стапель, Елизавета. «Системы линейных уравнений, решаемые методом исключения Гаусса». Purplemath
Доступно с https: // www.purplemath.com/modules/systlin6.htm .
Дата обращения [Дата] [Месяц] 2016 г.

2.2: Системы линейных уравнений и метод Гаусса-Джордана

Цели обучения

В этом разделе вы узнаете о

  1. Представьте систему линейных уравнений в виде расширенной матрицы
  2. Решите систему, используя элементарные операции со строками.

В этом разделе мы научимся решать системы линейных уравнений, используя процесс, называемый методом Гаусса-Жордана.Процесс начинается с того, что сначала система выражается в виде матрицы, а затем сводится к эквивалентной системе с помощью простых операций со строками. Процесс продолжается до тех пор, пока решение не станет очевидным из матрицы. Матрица, которая представляет систему, называется расширенной матрицей , а арифметические операции, которые используются для перехода от системы к сокращенной эквивалентной системе, называются строковой операцией .

Пример \ (\ PageIndex {1} \)

Запишите следующую систему как расширенную матрицу.

\ [\ begin {array} {l}
2 x + 3 y-4 z = 5 \\
3 x + 4 y-5 z = -6 \\
4 x + 5 y-6 z ​​= 7
\ конец {массив} \ nonumber \]

Решение

Мы выражаем вышеуказанную информацию в матричной форме. Поскольку система полностью определяется своей матрицей коэффициентов и матрицей постоянных членов, расширенная матрица будет включать только матрицу коэффициентов и постоянную матрицу. Итак, расширенная матрица, которую мы получаем, выглядит следующим образом:

\ [\ left [\ begin {array} {ccc | c}
2 & 3 & -4 & 5 \\
3 & 4 & -5 & -6 \\
4 & 5 & -6 & 7
\ конец {массив} \ nonumber \ right] \ nonumber \]

В последнем разделе мы выразили систему уравнений как \ (AX = B \), где \ (A \) представляет матрицу коэффициентов, а \ (B \) — матрицу постоянных членов.В качестве расширенной матрицы мы записываем матрицу как \ (\ left [\ begin {array} {l | l} A & B \ end {array} \ right] \). Ясно, что вся информация сохраняется в этой матричной форме, и отсутствуют только буквы \ (x \), \ (y \) и \ (z \). Учащийся может написать \ (x \), \ (y \) и \ (z \) поверх первых трех столбцов, чтобы облегчить переход.

Пример \ (\ PageIndex {2} \)

Для следующей расширенной матрицы запишите систему уравнений, которую она представляет.

\ [\ left [\ begin {array} {ccccc}
1 & 3 & -5 & | & 2 \\
2 & 0 & -3 & | & -5 \\
3 & 2 & -3 & | & -1
\ end {array} \ right] \ nonumber \]

Решение

Легко получить систему, как показано ниже.

\ [\ begin {array} {l}
x + 3 y-5 z = 2 \\
2 x-3 z = -5 \\
3 x + 2 y-3 z = -1
\ end { массив} \ nonumber \]

После того, как система выражена как расширенная матрица, метод Гаусса-Жордана сокращает ее до ряда эквивалентных систем, используя операции со строками. Это сокращение строк продолжается до тех пор, пока система не будет выражена в так называемой сокращенной форме эшелона строк . Уменьшенная ступенчатая форма матрицы коэффициентов имеет единицы по главной диагонали и нули в других местах.Решение легко получить из этой формы.

Этот метод не сильно отличается от алгебраических операций, которые мы использовали в методе исключения в первой главе. Основное отличие состоит в том, что он носит алгоритмический характер и, следовательно, может быть легко запрограммирован на компьютере.

Далее мы решим систему двух уравнений с двумя неизвестными, используя метод исключения, а затем покажем, что этот метод аналогичен методу Гаусса-Жордана.

Пример \ (\ PageIndex {3} \)

Решите следующую систему методом исключения.

\ [\ begin {array} {l}
x + 3 y = 7 \\
3 x + 4 y = 11
\ end {array} \ nonumber \]

Решение

Умножаем первое уравнение на — 3 и прибавляем его ко второму уравнению.

\ begin {align}
-3 x-9 y & = — 21 \\
3 x + 4 y & = 11 \\ \ hline
-5y & = — 10
\ end {align}

Таким образом мы преобразовали нашу исходную систему в эквивалентную систему:

\ begin {выравнивается}
x + 3 y & = 7 \\
-5 y & = — 10
\ end {выравнивается}

Делим второе уравнение на — 5, и получаем следующую эквивалентную систему.

\ begin {выравнивается}
x + 3 y & = 7 \\
y & = 2
\ end {выравнивается}

Теперь мы умножаем второе уравнение на — 3 и прибавляем к первому, получаем

\ [\ begin {array} {l}
x = 1 \\
y = 2
\ end {array} \ nonumber \]

Пример \ (\ PageIndex {4} \)

Решите следующую систему из примера 3 методом Гаусса-Жордана и покажите сходство в обоих методах, написав уравнения рядом с матрицами.

\ begin {array} {l}
x + 3 y = 7 \\
3 x + 4 y = 11
\ end {array}

Решение

Расширенная матрица для системы выглядит следующим образом.

\ [\ left [\ begin {array} {cccc}
1 & 3 & | & 7 \\
3 & 4 & | & 11
\ end {array} \ right] \ quad \ left [\ begin {array} {c}
x + 3 y = 7 \\
3 x + 4 y = 11
\ end {array} \ right] \ nonumber \]

Умножаем первую строку на — 3, и прибавляем ко второй строке.

\ [\ left [\ begin {array} {cccc}
1 & 3 & | & 7 \\
0 & -5 & | & -10
\ end {array} \ right] \ quad \ left [\ begin {array} {c}
x + 3 y & = 7 \\
-5 y & = — 10
\ end {array} \ right] \ nonumber \]

Вторую строку делим на — 5, получаем,

\ [\ left [\ begin {array} {llll}
1 & 3 & | & 7 \\
0 & 1 & | & 2
\ end {array} \ right] \ quad \ left [\ begin {array} {rl}
x + 3 y & = 7 \\
y & = 2
\ end {array} \ right] \ nonumber \]

Наконец, мы умножаем вторую строку на -3 и прибавляем к первой строке, и мы получаем

.

\ [\ left [\ begin {array} {llll}
1 & 0 & | & 1 \\
0 & 1 & | & 2
\ end {array} \ right] \ quad \ left [\ begin {array} {l}
x = 1 \\
y = 2
\ end {array} \ right] \ nonumber \]

Теперь мы перечислим три операции со строками, которые использует метод Гаусса-Жордана.

Операции со строками

  1. Любые две строки в расширенной матрице можно поменять местами.
  2. Любая строка может быть умножена на ненулевую константу.
  3. Постоянное кратное одной строке может быть добавлено к другой строке.

Легко видеть, что эти операции с тремя строками могут изменить внешний вид системы, но они не меняют решения системы.

Операция первой строки утверждает, что если любые две строки системы меняются местами, полученная новая система имеет то же решение, что и старая.Давайте посмотрим на пример в двух уравнениях с двумя неизвестными. Рассмотрим систему

\ begin {выравнивается}
x + 3 y & = 7 \\
3 x + 4 y & = 11
\ end {выравнивается}

Меняем ряды местами, и получаем,

\ begin {выравнивается}
3 x + 4 y & = 11 \\
x + 3 y & = 7
\ end {выравнивается}

Очевидно, что эта система имеет то же решение, что и предыдущая.

Вторая операция утверждает, что если строка умножается на любую ненулевую константу, полученная новая система имеет то же решение, что и старая.Снова рассмотрим указанную выше систему,

\ begin {выравнивается}
x + 3 y & = 7 \\
3 x + 4 y & = 11
\ end {выравнивается}

Умножаем первую строку на –3, получаем

\ begin {выравнивается}
-3 x-9 y & = — 21 \\
3 x + 4 y & = 11
\ end {выравнивается}

Опять же, очевидно, что эта новая система имеет то же решение, что и исходная.

Операция третьей строки утверждает, что любое постоянное кратное одной строки, добавленной к другой, сохраняет решение.Рассмотрим нашу систему,

\ begin {выравнивается}
x + 3 y & = 7 \\
3 x + 4 y & = 11
\ end {выравнивается}

Если мы умножим первую строку на –3 и прибавим ее ко второй строке, мы получим

\ begin {выравнивается}
x + 3 y & = 7 \\
-5 y & = — 10
\ end {выравнивается}

И снова сохраняется то же самое решение.

Теперь, когда мы понимаем, как работают операции с тремя строками, пора ввести метод Гаусса-Жордана для решения систем линейных уравнений.Как упоминалось ранее, метод Гаусса-Жордана начинается с расширенной матрицы и с помощью серии операций со строками заканчивается матрицей, которая находится в форме сокращенного эшелона строк .

Матрица находится в сокращенном эшелоне строк формирует , если первая ненулевая запись в каждой строке равна 1, а столбцы, содержащие эти единицы, имеют все остальные записи как нули. Форма сокращенного эшелона строк также требует, чтобы ведущая запись в каждой строке была справа от ведущей записи в строке над ней, а строки, содержащие все нули, были перемещены вниз.Сформулируем метод Гаусса-Жордана следующим образом.

Метод Гаусса-Джордана

  1. Запишите расширенную матрицу.
  2. Поменяйте местами строки, если необходимо, чтобы получить ненулевое число в первой строке, первом столбце.
  3. Используйте строковую операцию, чтобы получить 1 в качестве записи в первой строке и первом столбце.
  4. Используйте операции со строками, чтобы сделать все остальные записи нулями в первом столбце.
  5. Поменяйте местами строки, если необходимо, чтобы получить ненулевое число во второй строке, втором столбце.Используйте строковую операцию, чтобы сделать эту запись 1. Используйте строковую операцию, чтобы сделать все остальные записи нулями во втором столбце.
  6. Повторите шаг 5 для строки 3, столбца 3. Продолжайте двигаться по главной диагонали, пока не дойдете до последней строки или пока число не станет равным нулю.

Итоговая матрица называется сокращенной строкой-эшелонированной формой.

Пример \ (\ PageIndex {5} \)

Решите следующую систему методом Гаусса-Жордана.

\ begin {выравнивается}
2 x + y + 2 z & = 10 \\
x + 2 y + z & = 8 \\
3 x + y-z & = 2
\ end {выравнивается}

Решение

Пишем расширенную матрицу.

\ [\ left [\ begin {array} {ccccc}
2 & 1 & 2 & | & 10 \\
1 & 2 & 1 & | & 8 \\
3 & 1 & -1 & | & 2
\ end {array} \ right] \ nonumber \]

Нам нужна 1 в первой строке, первом столбце. Этого можно добиться, разделив первую строку на 2 или поменяв местами вторую строку с первой. Перестановка строк — лучший выбор, потому что таким образом мы избегаем дробей.

\ [\ left [\ begin {array} {ccccc}
1 & 2 & 1 & | & 8 \\
2 & 1 & 2 & | & 10 \\
3 & 1 & -1 & | & 2
\ end {array} \ right] \ quad \ text {мы поменяли местами строку 1 (R1) и строку 2 (R2)} \ nonumber \]

Нам нужно обнулить все остальные записи в столбце 1.Чтобы сделать запись (2) нулем в строке 2, столбце 1, мы умножаем строку 1 на — 2 и добавляем ее во вторую строку. Получаем,

\ [\ left [\ begin {array} {ccccc}
1 & 2 & 1 & | & 8 \\
0 & -3 & 0 & | & -6 \\
3 & 1 & -1 & | & 2
\ end {array} \ right] \ quad-2 R 1 + R 2 \ nonumber \]

Чтобы сделать запись (3) нулем в строке 3, столбце 1, мы умножаем строку 1 на — 3 и добавляем ее в третью строку. Получаем,

\ [\ left [\ begin {array} {ccccc}
1 & 2 & 1 & | & 8 \\
0 & -3 & 0 & | & -6 \\
0 & -5 & -4 & | & -22
\ end {array} \ right] \ quad-3 R 1 + R 3 \ nonumber \]

Пока что мы поставили 1 в левом углу и все остальные записи в этом столбце равны нулю.Теперь мы переходим к следующей диагональной записи, строке 2, столбцу 2. Нам нужно сделать эту запись (–3) равной 1 и обнулить все остальные записи в этом столбце. Чтобы сделать запись строки 2, столбца 2 равной 1, мы делим всю вторую строку на –3.

\ [\ left [\ begin {array} {ccccc}
1 & 2 & 1 & | & 8 \\
0 & 1 & 0 & | & 2 \\
0 & -5 & -4 & | & -22
\ end {array} \ right] \ quad \ mathrm {R} 2 \ div (-3) \ nonumber \]

Затем мы обнуляем все остальные записи во втором столбце.

\ [\ left [\ begin {array} {ccccc}
1 & 0 & 1 & | & 4 \\
0 & 1 & 0 & | & 2 \\
0 & 0 & -4 & | & -12
\ end {array} \ right] \ quad-2 R 2 + R 1 \ text {и} 5 R 2 + R 3 \ nonumber \]

Сделаем последнюю диагональную запись равной 1, разделив строку 3 на — 4.

\ [\ left [\ begin {array} {ccccc}
1 & 0 & 1 & | & 4 \\
0 & 1 & 0 & | & 2 \\
0 & 0 & 1 & | & 3
\ end {array} \ right] \ quad \ quad R 3 \ div (-4) \ nonumber \]

Наконец, мы обнуляем все остальные записи в столбце 3.

\ [\ left [\ begin {array} {ccccc}
1 & 0 & 0 & | & 1 \\
0 & 1 & 0 & | & 2 \\
0 & 0 & 1 & | & 3
\ end {array} \ right] \ quad- \ mathrm {R} 3+ \ mathrm {R} 1 \ nonumber \]

Ясно, что решение читается как \ (x = 1 \), \ (y = 2 \) и \ (z = 3 \).

Прежде чем мы закончим этот раздел, мы упомянем некоторые термины, которые могут нам понадобиться в четвертой главе.

Процесс получения 1 в местоположении с последующим обнулением всех остальных записей в этом столбце называется поворотом .

Число, равное 1, называется поворотным элементом , , , а строка, которая содержит поворотный элемент, называется поворотной строкой .

Мы часто умножаем сводную строку на число и добавляем ее к другой строке, чтобы получить в последней ноль. Строка, к которой добавлено кратное сводной строке, называется целевой строкой .

Как использовать метод исключения Гаусса для решения системы уравнений?

ПРИМЕР:

Используйте метод исключения Гаусса для решения следующей системы уравнений.

# x + 2y + 3z = -7 #
# 2x-3y-5z = 9 #
# -6z-8y + z = -22 #

Раствор:

Настроить расширенную матрицу формы.

# ((1,2,3, |, -7), (2,3, -5, |, 9), (- 6, -8,1, |, 22)) #

Цель 1. Получите 1 в верхнем левом углу.

Уже сделано.

Цель 2a: Получите ноль под единицей в первом столбце.

Умножьте строку 1 на # -2 #, чтобы получить

# ((- 2, -4, -6, |, 14)) #

Добавьте результат в строку 2 и поместите результат в строку 2.

Обозначим операции как # -2R_2 + R_1 → R_2 #.

# ((1,2,3, |, -7), (2,3, -5, |, 9), (- 6, -8,1, |, 22)) stackrel (-2R_1 + R_2 → R_2) (→) ((1,2,3, |, -7), (0, -7, -11, |, 23), (- 6, -8,1, |, 22)) #

Цель 2b: Получите еще один ноль в первом столбце.

Для этого нам понадобится операция # 6R_1 + R_3 → R_3 #.

# ((1,2,3, |, -7), (0, -7, -11, |, 23), (- 6, -8,1, |, 22)) stackrel (6R_2 + R_3 → R_3) (→) ((1,2,3, |, -7), (0, -7, -11, |, 23), (0,4,19, |, -64)) #

Цель 2c. Получите оставшийся ноль.

Умножьте строку 2 на # -1 / 7 #.

# ((1,2,3, |, -7), (0, -7, -11, |, 23), (0,4,19, |, -64)) stackrel (- (1/7 ) R_2 → R_2) (→) ((1,2,3, |, -7), (0,1,11 / 7, |, -23 / 7), (0,4,19, |, -64 )) #

Теперь используйте операцию # -4R_2 + R_3 → R_3 #.

# ((1,2,3, |, -7), (0,1,11 / 7, |, -23 / 7), (0,4,19, |, -64)) stackrel (-4R_2 + R_3 → R_3) (→) ((1,2,3, |, -7), (0,1,11 / 7, |, -23 / 7), (0,0,89 / 7, |, -356/7)) #

Умножьте третью строку на # 7/89 #.

# ((1,2,3, |, -7), (0,1,11 / 7, |, -23 / 7), (0,0,89 / 7, |, -356 / 7)) stackrel (7 / 89R_3 → R_3) (→) ((1,2,3, |, -7), (0,1,11 / 7, |, -23 / 7), (0,0,1, | , -4)) #

Цель 3. Используйте обратную подстановку, чтобы получить значения # x #, # y # и # z #.

Цель 3а. Рассчитайте # z #.

#z = -4 #

Цель 3b. Рассчитайте # y #.

# y + 11 / 7z = -23 / 7 #
# y-44/7 = -23 / 7 #
# y = 44 / 7-23 / 7 = 21/7 #

# у = 3 #

Цель 3c. Вычислить x.

# x + 2y + 3z = -7 #
# x + 6-12 = -7 #
# x-6 = -7 #

# х = 1 #

Решение: # x = 1, y = 3, z = -4 #

Как решить линейные системы с помощью исключения Гаусса — видео и стенограмма урока

Расширенная матрица

Помните, что матрица — это просто прямоугольный массив значений, помещенных в строки и столбцы.Сначала нам нужно превратить нашу линейную систему в матричную форму, превратив ее в расширенную матрицу. Расширенная матрица представляет собой комбинацию двух матриц. В нашем случае у нас есть матрица для коэффициентов левой части уравнения и еще одна матрица для правой части уравнения.

Напомним, что преобразование системы уравнений в матричную форму включает в себя выделение только коэффициентов вместе с их соответствующими знаками после их организации так, чтобы член x сначала сопровождался членом y , а затем членом z , знак равенства, а затем константа.Мы можем использовать вертикальную линию или несколько точек на вертикальной линии, чтобы обозначить наш знак равенства. Наша линейная система уже организована должным образом, поэтому все, что нам нужно сделать, это выделить наши коэффициенты. В нашей первой строке будет 1, 1, 1, | а затем 5. Во второй строке 2, 0, -1, | и 4. В нашей третьей строке 0, 3, 1, | и 2. Наша матрица выглядит так:

Исключение Гаусса

Теперь мы можем использовать метод исключения Гаусса, чтобы помочь нам решить эту линейную систему.Исключение Гаусса — это манипулирование расширенной матрицей до тех пор, пока мы не получим матрицу, которая представляет левую часть уравнений в верхней треугольной форме . Это означает, что мы хотим, чтобы все нули находились ниже главной диагонали. Эта главная диагональ начинается в верхнем левом углу и заканчивается в правом нижнем углу матрицы коэффициентов. Другими словами, мы хотим манипулировать матрицей так, чтобы 2 во второй строке и 0 и 3 в третьей строке были равны 0.

Чтобы заменить эти числа на нули, мы будем использовать наши операции со строками матрицы.Чтобы превратить наши первые 2 в 0, мы умножаем нашу первую строку на -2, а затем добавляем ее ко второй строке, чтобы создать новую вторую строку. Получаем новую вторую строку 0, -2, -3, | и -6. Теперь, чтобы заменить 3 в третьей строке на 0, мы будем использовать эту новую вторую строку в сочетании с третьей строкой. Мы умножим вторую строку на 3 и прибавим ее к третьей строке, умноженной на 2. Мы получим новую третью строку из 0, 0, -7, | и -14.

Мы не переписывали первую строку, потому что нам не нужно менять это уравнение. Мы только умножили первую строку на 3, чтобы вычесть ее из второй строки. Помните, что в алгебре всякий раз, когда мы умножаем уравнение на любую константу, уравнение не меняется вообще; цифры просто увеличиваются в разы.

Теперь, когда у нас есть нули под главной диагональю, мы закончили с помощью исключения Гаусса. Теперь мы можем продолжить и решить нашу линейную систему.

Решение системы

Обратите внимание, как легко теперь решить эту проблему. Если мы выпишем наши линейные уравнения, мы получим x + y + z = 5, -2 y — 3 z = -6 и -7 z = -14.Мы можем немедленно решить третье уравнение для z , чтобы получить z = -14 / -7 = 2. Затем мы можем подставить это значение для z во второе уравнение, чтобы найти следующую переменную, y . Получаем -2 y — 3 (2) = -6. Это превращается в -2 y — 6 = -6. Чтобы решить для y , мы прибавляем 6 к обеим сторонам и получаем -2 y = 0. Разделив обе стороны на -2, мы получаем y = 0. Итак, теперь у нас есть y = 0 и z = 2.Чтобы решить нашу последнюю переменную, x , мы можем использовать наше самое первое уравнение. Подставляя эти два значения, мы получаем x + 0 + 2 = 5. Решая это для x , мы получаем x = 3. Итак, наш окончательный ответ: x = 3, y = 0, и z = 2. Мы также можем записать это в точечной форме, например: (3, 0, 2).

Резюме урока

Давайте рассмотрим то, что мы узнали. Мы узнали, что линейных системы представляют собой наборы линейных уравнений.Линейная система имеет такое же количество уравнений, как и переменных, поскольку нам нужно по одному уравнению для каждой переменной, чтобы решить такую ​​систему уравнений. В методе, о котором мы говорили в этом уроке, используется метод исключения Гаусса , метод решения системы уравнений, который включает манипулирование матрицей так, чтобы все элементы ниже главной диагонали равнялись нулю. Верхний треугольник — это термин, используемый для описания матрицы, в которой все нули находятся ниже главной диагонали. Затем мы используем алгебру и подстановку, чтобы завершить решение нашей системы уравнений.

Результаты обучения

Завершение этого урока может подготовить вас к:

  • Чтению определений линейных систем, расширенной матрицы и верхней треугольной формы
  • Продемонстрировать использование метода исключения Гаусса для решения системы уравнений

ИСКЛЮЧЕНИЕ ПО ГАУССУ: РЕШЕНИЕ СИСТЕМ ЛИНЕЙНЫХ УРАВНЕНИЙ: ПРИМЕРЫ И РЕШАЕМЫЕ ЗАДАЧИ: СТАРШАЯ ШКОЛА

Содержимое этой страницы:


Введение

Система уравнений (линейная) — это группа (линейных) уравнений с различные неизвестные факторы.Вообще говоря, неизвестные факторы входят в различные уравнения.

Уравнение с различными неизвестными факторами связывает их друг с другом.

Решение системы состоит в нахождении значения неизвестных факторов способом, который проверяет все уравнения, составляющие систему.

  • Если существует одно решение (одно значение для каждого неизвестного фактора), мы будем говорить, что система Согласованная независимая система (СНГ) .

  • Если существуют различные решения (система имеет бесконечно много решений), мы говорим, что система является Согласованная зависимая система (CDS). .

  • Если решения нет, а это произойдет, если есть два или несколько уравнений, которые нельзя проверить одновременно, мы говорим, что это несогласованная система (IS) .Например, следующая система уравнений

    $$ \ begin {case} \ begin {array} {lcl} y & = & 0 \\ 2x + y & = & 0 \\ 2x + y & = & 2 \ end {array} \ end {ases} $ $

    несовместимо, потому что мы получаем решение x = 0 из второго уравнения и, из третьего, x = 1 .

В этом разделе мы собираемся решать системы, используя метод исключения Гаусса , который заключается в простом выполнении элементарных операций в строке или столбце расширенной матрицы, чтобы получить свой эшелон из или его сокращенный эшелон форма (Гаусс-Джордан).


Метод разрешения

  1. Применяем метод исключения Гаусса-Джордана : получаем уменьшенная строка эшелона формирует из расширенной матрицы систему уравнений, выполняя элементарные операции в строках (или столбцах).

  2. Когда у нас есть матрица, мы применяем теорему Руше-Капелли , чтобы определить тип системы и получить решение (я), а именно:

    Пусть A · X = B будет системой m линейных уравнений с n неизвестными факторы, m и n натуральные числа (не ноль):

    • AX = B соответствует тогда и только тогда, когда,

      $$ ранг (A) = ранг (A | B) $$

    • AX = B является непротиворечивым независимым тогда и только тогда, когда,

      $$ ранг (A) = n = ранг (A | B) $$

Примечание: Элементарные операции в строках или столбцах позволяют получить системы, эквивалентные исходной, но с формой, упрощающей получение решений (если они есть). Также есть более быстрые инструменты для выработки решений в СНГ, такие как правило Крамера.




Система 1

Показать решение

Расширенная матрица системы

того же размера, что и система (2×3). Вертикальная линия, отделяющая матричные коэффициенты от вектора независимых членов.

Выполним элементарные операции в строках, чтобы получить сокращенную форму эшелона строк:

Умножаем первую строку на 1/5 а вторую на 1/3

Добавляем вторую строку с первой

Вторую строку умножаем на 5/7

Складываем первую строку со второй, умноженной на -2/5

Эта последняя эквивалентная матрица представлена ​​в сокращенной форме эшелона строк. и это позволяет нам быстро увидеть рейтинг матрица коэффициентов и дополненная.

Считаем ранги:

По теореме Руше-Капелли система непротиворечива Независимая. Полученная матрица представляет собой систему

, который является решением исходной системы.



Система 2

Показать решение

Расширенная матрица системы

Проводим элементарные операции в строках для получения приведенных форма эшелона строки:

Вторую строку умножаем на 1/2

Добавляем первую строку со второй

Умножаем первую строку на 1/3

Эта последняя эквивалентная матрица находится в форме сокращенного эшелона строк и имеет нулевую строку, что означает, что строки в исходной системе линейно зависимы. (любой из них может быть получен путем умножения другого на скаляр, не равный нулю).

Рассчитываем ранги

По теореме Руше-Капелли система непротиворечива. Кроме того, это зависит, потому что ранг (1) ниже, чем количество неизвестных факторов (2).

Полученная матрица представляет собой систему

Решения



Система 3

Показать решение

Расширенная матрица системы

Выполняем элементарные операции в строках, чтобы получить сокращенную форму эшелона строк

Мы меняем порядок строк (так у нас уже будет 1 в первой строке без необходимости умножать)

Складываем вторую строку с первой, умноженной на 5 :

Вторую строку умножаем на -1/15

Эта последняя матрица имеет эшелонированную форму (не сокращена).

Мы можем непосредственно заметить, что система несовместима, потому что у нас следующее равенство (вторая строка):

$$ 0x + 0y = 1 $$

, что невозможно равенство.

Рассчитываем ранги матрицы коэффициентов и дополненной:

По теореме Руше-Капелли система несовместна (решения нет). Полученная матрица представляет собой систему

Система непоследовательна, потому что мы имеем невозможное равенство

$$ 0 = 1 $$



Система 4

Показать решение

Расширенная матрица системы

(размер 3х4).

Выполняем элементарные операции в строках, чтобы получить сокращенную форму эшелона строк

Умножаем первую строку на 1/5

Складываем вторую и третью строки с первой, умноженной на -2

Вторую и третью строки умножаем на 5

Складываем вторую строку с третьей, умноженной на -1

Умножаем вторую строку на -1/10 и третью на 1/11

Складываем первую строку со второй, умноженной на -2/5 , и третью со второй, умноженной на -1

Третью строку умножаем на -11/5

Эта последняя эквивалентная матрица находится в сокращенной форме эшелона строк (мы знаем ее, потому что это единичная матрица). Имея единичную матрицу, мы знаем, что это непротиворечивая независимая система, и можем получить единственное решение.

Рассчитываем ранги

По теореме Руше-Капелли система непротиворечива Независимая. Полученная матрица представляет собой систему

, который является решением системы.



Система 5

Показать решение

Расширенная матрица системы

Выполняем элементарные операции в строках, чтобы получить сокращенную форму эшелона строк

Мы вычитаем первую строку из второй и добавляем третью строку с первой.

Умножаем первую строку на 1/2 , а вторую на 1/3

Мы складываем первую строку со второй, умноженной на 1/2 , и третью строку со второй, умноженной на -2

Третью строку умножаем на 1/3

Складываем первую строку с третьей, умноженной на -3/2

Эта последняя матрица представляет собой эшелонированную матрицу с сокращенным числом строк (мы знаем это, потому что у нас есть единичная матрица).

Рассчитываем ранги

По теореме Руше-Капелли система непротиворечива Независимая. Полученная матрица представляет собой систему



Система 6

Показать решение

Расширенная матрица системы

Выполняем элементарные операции в строках, чтобы получить сокращенную форму эшелона строк

Умножаем вторую строку на -1/3 а третью на 1/4

Складываем вторую и третью строки, умножив первую на -1

Умножаем вторую строку на -3/4 а третью на -1

Третью строку складываем со второй, умноженной на -1

Эта последняя матрица имеет эшелонированную форму (но не сокращена) и мы не продолжаем выполнять операции по строкам, потому что можем видим, что последняя строка делает систему непоследовательной. Эта строка сообщает нам:

$$ 0x + 0y + 0z = -1 $$

И все это невозможное равенство.

Фактически у нас есть ранги

По теореме Руше-Капелли система несовместна.



Система 7

Показать решение

Расширенная матрица системы

Выполняем элементарные операции в строках, чтобы получить сокращенную форму эшелона строк

Умножаем первую и вторую строки на 1/3 и на 1/6 соответственно

Вторую строку вычитаем из третьей

Первый ряд складываем со вторым

Третью строку умножаем на 3

Эта последняя матрица находится в сокращенном эшелоне строк формы , поэтому мы можем легко вычислить ранги:

Рассчитываем ранги

По теореме Руше-Капелли система непротиворечива. Но он не является независимым, потому что количество неизвестных факторов (3) отличается от ранга. Полученная матрица представляет собой систему

Решения



Система 8

Показать решение

Расширенная матрица системы

Примечание: прежде, чем мы начнем, мы должны прокомментировать, что процедура будет таким же, как и было до сих пор.Но у нас есть корни в матрице, а это значит, что операции над строками будут длинными и утомительно. Эта задача не очень интересна в дидактическом смысле, помимо расчетов.

Проводим элементарные операции в строках, чтобы получить пониженный рядный эшелон формы

Мы складываем вторую строку с первой, умноженной на -√5 и, с третья, умноженная на -2/5

Умножаем вторую строку на 1 / √5 и третью на 5/17

К первой прибавляем третью и ко второй вычитаем третью

Переписываем матрицу

Вторую строку умножаем на (√5 / 5-5) -1

Складываем первую строку со второй, умноженной на -5

В этой последней матрице (почти) в строке уменьшено форма эшелона (надо поменять второй и третий ряды так что это правда). Из последней матрицы получаем решения:



Система 9

Показать решение

Расширенная матрица системы

Выполняем элементарные операции в строках, чтобы получить сокращенную форму эшелона строк

Складываем третью и четвертую строки, умножая первую на 3 и -2 соответственно

Складываем первую, вторую и третью строки, четвертую умножаем на 2 , -3 и 5 соответственно

Четвертую строку умножаем на -1 и меняем на вторую

Умножаем третью и четвертую строки на 1/34 и -1/22 соответственно

Вычитаем третью строку из четвертой

Четвертую умножаем на -187/42

К первой строке прибавляем третью умноженную на -13 и вторую на 8

К первой строке прибавляем четвертую, умноженную на 5/34 , ко второй прибавляем первую, умноженную на 5/17 и к третьему добавляем первое, умноженное на -3/34

По теореме Руше-Капелли система непротиворечива Независима, и решение равно



Система 10

Показать решение

Расширенная матрица системы

Примечание: Эта система была включена в объективно отметить, что теория матрицы применима к комплексным числам. Единственное отличие от предыдущих систем в том, что теперь нам нужно действовать путем умножения и деления комплексные числа.

Выполняем элементарные операции в строках для получения приведенной формы эшелона строк

Умножаем вторую строку на 1/2 и меняем ее на первую

Добавляем вторую строку с первой, умноженной на — (1 + i)

Вторую строку умножаем на

Добавляем первую строку ко второй умноженной на -i / 2

Эта последняя матрица имеет сокращенную форму, поскольку она является единичной матрицей.

По теореме Руше-Капелли система непротиворечива Независима, и решение равно



Matesfacil.com от J. Llopis под лицензией творческий Международная лицензия Commons Attribution-NonCommercial 4. 0.


Часть 6: Исключение по Гауссу. Исключение Гаусса — это алгоритм… | Авниш | Линейная алгебра

Метод исключения Гаусса — это алгоритм решения системы линейных уравнений.Он назван в честь немецкого математика Карла Фридриха Гаусса.

Карл Фридрих Гаусс

Аналогичен методу исключения, описанному ранее.

Для выполнения метода исключения Гаусса:

  1. Создаем расширенную матрицу коэффициентов и констант данной системы линейных уравнений.
  2. Выбираем нашу pivot (это первый элемент по диагонали). Затем мы пытаемся уменьшить все элементы под ним (до «0»), используя pivot.

Мы делаем это, выполняя два вида операций:

a) Умножение сводной строки (строки сводного элемента) на скалярную величину и вычитание или добавление ее строк под ней.

b) Перестановка строк (например, строка 2 заменяется строкой 3)

Затем мы выбираем следующую точку поворота (следующий элемент по диагонали) и уменьшаем элементы под ней.

3. Разбиваем расширенную матрицу обратно на строковое изображение и выполняем умножение с переменной матрицей. Получаем новые редуцированные уравнения.

Мы решаем эти уравнения, чтобы получить значения неизвестных (переменных).

Предполагая, что мы должны найти решение (я) следующей системы уравнений:

4x + y = 9 → (1)

2x-y = 3 → (2)

5x-3y = 7 → ( 3)

(пример «Одно уникальное решение» из Части 5)

Шаг 1 (Создание расширенной матрицы):

Для выполнения исключения Гаусса мы берем изображение строки (1), (2) и (3) .Это будет выглядеть следующим образом:

Затем мы создаем расширенную матрицу для матрицы коэффициентов и постоянной матрицы.

Единая матрица со значениями коэффициентов и констант, разделенных пунктирной линией

Шаг 2 (Исключение):

Шаг 2A:

Принимая элемент в верхнем левом углу (первый элемент по диагонали) в качестве стержня, мы стремимся исключить ( уменьшите до «0») все элементы под ним. Другими словами, мы должны преобразовать каждый элемент в столбце 1 в «0», кроме pivot.

Элемент поворота будет выделен красным цветом, а элементы, которые необходимо исключить, — синим.

Итак, мы умножаем первую строку на скаляр 1/2 и вычитаем ее из второй строки.

Элемент в строке 2 и столбце 1 исключается.

Затем мы умножаем первую строку на скаляр 5/4 и вычитаем из третьей строки.

Элемент в строке 3 и столбце 1 исключен.

Теперь все элементы в первом столбце равны «0», кроме точки поворота.

Шаг 2B:

Теперь следующий элемент по диагонали (второй столбец второй строки) установлен как опорный, и мы стремимся удалить все элементы под ним.

Pivot выделен красным.

Итак, мы умножаем вторую строку на скаляр 17/6 и вычитаем ее из третьей строки.

Элемент в строке 3 и столбце 2 исключен.

Результат — верхняя треугольная матрица.

Текущее состояние расширенной матрицы называется строкой эшелона формы .

Шаг 3 (обратная подстановка):

Теперь мы конвертируем форму эшелона строки обратно в изображение строки.

У нас было аналогичное уравнение на этапе 1

При умножении мы получаем:

Мы составляем уравнения из этих

4x + y = 9 → (4)

-3y / 2 = -3/2 → (5)

Решая (5) относительно «y», получаем:

y = 1

Теперь подставляем y = 1 в (4):

4x + 1 = 9

4x = 8

x = 2

Итак, мы получаем x = 2 и y = 1, именно то, что мы получили, когда решали через изображение строки и изображение столбца в Части 5.

Теперь применим тот же алгоритм еще в двух случаях (бесконечно много решений и нет решения).

Бесконечно много решений

Мы возьмем тот же пример, что и в Части 5. То есть:

x + 2y = 4 → (6)

2x + 4y = 8 → (7)

Шаг 1 (Создание дополненного матрица):

Изображение строки (6) и (7) Расширенная матрица изображения строки выше

Шаг 2 (Исключение):

Первый элемент — диагональный («1») в качестве точки поворота.

Pivot выделен красным, и мы должны удалить все элементы под ним (синим). Чтобы исключить «2», мы дважды вычитаем строку 1 из строки 2 Теперь последняя строка полностью заполнена 0

Мы больше не делаем поворота так как исключать нечего.

Шаг 3 (обратная подстановка):

Мы преобразуем форму эшелона строк обратно в изображение строки:

После этого мы умножаем ее и получаем новые уравнения

x + 2y = 4 → (8)

Уравнение (6) и уравнения (8) такие же, и у нас есть только одно уравнение после исключения, но два неизвестных («x» и «y»).

Существует множество значений, которыми можно заменить x и y, чтобы удовлетворить (8).

Нравится, x = 0 и y = 2. Подставляя в уравнение (8), получаем:

0 + 2 (2) = 4

4 = 4

Или x = 1 и y = 1.5. Подставляя в уравнение (8), получаем:

1 + 2 (1.5) = 4

1 + 3 = 4

4 = 4

Таким образом, система уравнений (6) и (7) имеет бесконечно много решений.

Нет решения

Рассмотрим систему линейных уравнений следующим образом:

x + y = 4 → (9)

x + y = 8 → (10)

xy = 0 → (11)

Применение гауссовского Устранение.

Шаг 1 (Создание расширенной матрицы):

Изображение строки (9), (10) и (11) Расширенная матрица коэффициентов и констант

Шаг 2 (Исключение):

Принятие первого диагонального элемента в качестве точки поворота

Мы выполняем следующие две операции:

и матрица, которую мы получаем:

У нас все еще нет формы эшелона строк (верхняя треугольная матрица).

Итак, мы выполняем обмен строк (который также является вариантом на этапе исключения в методе исключения по Гауссу):

Замена строки 3 на строку 2 Форма эшелона строк

Шаг 3 (обратная подстановка):

Форма эшелона строк преобразована обратно в изображение строки

Уравнения, которые мы получаем после умножения матриц выше:

x + y = 4 → (12)

-2y = -4 → (13)

Решая уравнение (13) относительно «y», получаем:

y = 2

Подставляя y = 2 в уравнение (12), мы получаем:

x + 2 = 4

x = 2

Чтобы подтвердить, что x = 2 и y = 2 является решением, мы подставляем их в систему уравнений i. е. (9), (10) и (11).

Подставляя в (9), получаем:

2 + 2 = 4

4 = 4

x = 2 и y = 2, удовлетворяет (9).

Подставляя в (10), получаем:

2 + 2 = 8

4 ≠ 8, это не удовлетворяет (10).

Следовательно, x = 2 и y = 2 не является решением (9), (10) и (11), и не существует решения этой системы линейных уравнений, как мы видели в прошлой статье.

Одно решение

Когда количество неизвестных (переменных) равно количеству уравнения в системе линейных уравнений.

В качестве примера (1), (2) и (3):

4x + y = 9 → (1)

2x-y = 3 → (2)

5x-3y = 7 → (3)

Имеется 2 неизвестных («x» и «y») и 3 уравнения ((1), (2) и (3)).

Двух уравнений было бы достаточно для двух неизвестных.

Бесконечно много решений

Когда количество неизвестных превышает количество уравнений.

В примере (6) и (7):

x + 2y = 4 → (6)

2x + 4y = 8 → (7)

Есть 2 неизвестных («x» и «y») и 2 уравнения ((6) и (7)).